Exam 1 Prep U Flashcards

1
Q

The nurse has received a client into care who was admitted with a heroin overdose. The client has a 5-year history of illicit substance use with cocaine, heroine and oxycodone. The client develops a sudden onset of wheezing, restlessness and a cough that produces a frothy, pink sputum. The nurse suspects the client has most likely developed which complication of opioid overdose?

A

Pulmonary edema
Explanation:
The nurse should suspect the client has developed pulmonary edema, which is frequently seen in clients who abuse/overdose on narcotics. Many drugs — ranging from illegal drugs such as heroin and cocaine to aspirin — are known to cause noncardiogenic pulmonary edema. Pneumonia is not the likely cause given the sudden onset of respiratory symptoms accompanied but coughing up the pink frothy sputum. The client’s history of illicit substance use and now overdose on these drugs should lead the nurse to suspect pulmonary edema is the cause of the sudden onset of these symptoms over congestive heart failure, in which clients have a more gradual onset of respiratory issues. Although a panic attack can manifest in shortness or breath and restlessness, the client would not be wheezing or producing blood tinged sputum with a cough. Panic attacks do, however, have a sudden onset and can cause the client chest pain and a sense of doom.

How well did you know this?
1
Not at all
2
3
4
5
Perfectly
2
Q

For a patient who is experiencing multiple injuries, which sequence of medical or nursing management would the nurse identify as a priority?

A

Establish an airway, control hemorrhage, prevent hypovolemic shock, assess for head injuries.
Explanation:
The goals of treatment are to determine the extent of injuries and to establish priorities of treatment. Priority management includes 1) establishing an airway and ventilation, 2) controlling hemorrhage, 3) preventing hypovolemic shock, and 4) assessing for head and neck injuries.

How well did you know this?
1
Not at all
2
3
4
5
Perfectly
3
Q

A homeless client presents to the ED. Upon assessment, the client is experiencing hypothermia. The nurse will plan to complete which priority intervention during the rewarming process?

A

Attach a cardiac monitor
Explanation:
Continuous electrocardiograph (ECG) monitoring is performed during the rewarming process because cold-induced myocardial irritability leads to conduction disturbances, especially ventricular fibrillation. A urinary catheter should be inserted to monitor urinary output; however, ECG monitoring is the priority. There is no indication for endotracheal intubation. Inotropic medications are contraindicated because they can stimulate the heart and increase the risk for fatal dysrhythmias, such as ventricular fibrillation.

How well did you know this?
1
Not at all
2
3
4
5
Perfectly
4
Q

A backcountry skier has been airlifted to the ED after becoming lost and developing hypothermia and frostbite. How should the nurse best manage the client’s frostbite?

A

Immerse affected extremities in water slightly above normal body temperature.
Explanation:
Frozen extremities are usually placed in a 37°C to 40°C (98.6°F to 104°F) circulating bath for 30- to 40-minute spans. To avoid further mechanical injury, the body part is not handled. Massage is contraindicated.

How well did you know this?
1
Not at all
2
3
4
5
Perfectly
5
Q

A client is admitted to the ED after being involved in a motor vehicle accident. The client has multiple injuries. After establishing an airway and adequate ventilation, the ED team should prioritize what aspect of care?

A

Control the client’s hemorrhage.
Explanation:
After establishing airway and ventilation, the team should evaluate and restore cardiac output by controlling hemorrhage. This must precede neurologic assessments and treatment of skeletal injuries.

How well did you know this?
1
Not at all
2
3
4
5
Perfectly
6
Q

Permanent brain injury or death will occur within which time frame secondary to hypoxia?

A

3 to 5 minutes
Explanation:
If the airway is completely obstructed, permanent brain injury or death will occur within 3 to 5 minutes secondary to hypoxia. Air movement is absent in the presence of complete airway obstruction. Oxygen saturation of the blood decreases rapidly because obstruction of the airway prevents air from entering the lungs. Oxygen deficit occurs in the brain, resulting in unconsciousness, with death following rapidly. The other time frames are incorrect.

How well did you know this?
1
Not at all
2
3
4
5
Perfectly
7
Q

An 83-year-old client is brought in by ambulance from a long-term care facility. The client’s symptoms are weakness, lethargy, incontinence, and a change in mental status. The nurse knows that emergencies in older adults may be more difficult to manage for what reason?

A

Older adults may have an altered response to treatment.
Explanation:
Emergencies in this age group may be more difficult to manage because elderly clients may have an atypical presentation, an altered response to treatment, a greater risk of developing complications, or a combination of these factors. The elderly client may perceive the emergency as a crisis signaling the end of an independent lifestyle or even resulting in death. Stigmatization and nonadherence to treatment are not commonly noted. Older adults do not necessarily have difficulty giving a health history.

How well did you know this?
1
Not at all
2
3
4
5
Perfectly
8
Q

When preparing to perform abdominal thrusts on a client with an airway obstruction, which of the following would be most appropriate?

A

Positioning the hands in the midline slightly above the umbilicus
Explanation:
When performing abdominal thrusts, the nurse would place the thumb side of one fist against the client’s abdomen in the midline slightly above the umbilicus and well below the xiphoid process, grasping the fist with the other hand. Then the nurse would press the fist into the client’s abdomen with a quick inward and upward thrust such that each new thrust should be a separate and distinct maneuver. The unconscious client is positioned on the back. The client who is conscious should be standing or sitting.

How well did you know this?
1
Not at all
2
3
4
5
Perfectly
9
Q

A patient with frostbite to both lower extremities from exposure to the elements is preparing to have rewarming of the extremities. What intervention should the nurse provide prior to the procedure?

A

Administer an analgesic as ordered.
Explanation:
During rewarming, an analgesic for pain is administered as prescribed, because the rewarming process may be very painful. To avoid further mechanical injury, the body part is not handled. Massage is contraindicated.

How well did you know this?
1
Not at all
2
3
4
5
Perfectly
10
Q

A nurse is providing an educational program for a group of occupational health nurses working in chemical facilities. Which of the following would the nurse include as the priority in the case of a chemical burn?

A

Rinsing the area with copious amounts of water
Explanation:
The priority for any chemical burn is to immediately drench the area with running water, unless the chemical is lye or white phosphorus, which should be brushed off the patient. Antimicrobial ointments, sterile dressings, and tetanus prophylaxis are measures instituted later in the course of treatment, depending on the characteristics of the chemical agent and the size and location of the burn.

How well did you know this?
1
Not at all
2
3
4
5
Perfectly
11
Q

What is a common source of airway obstruction in an unconscious client?

A

The tongue
Explanation:
In an unconscious client, the muscles controlling the tongue commonly relax, causing the tongue to obstruct the airway. When this situation occurs, the nurse should use the head-tilt, chin-lift maneuver to cause the tongue to fall back into place. If she suspects the client has a neck injury she must perform the jaw-thrust maneuver.

How well did you know this?
1
Not at all
2
3
4
5
Perfectly
12
Q

A nurse is preparing to assist with a gastric lavage for a client who has ingested an unknown poison and is obtunded. To ensure that the tube reaches the stomach, the nurse would measure the distance from the bridge of the nose to which of the following?

A

Ear lobe and then to the xiphoid process
Explanation:
The nurse measures the tube from the bridge of the nose to the xiphoid process to ensure that the tube reaches the stomach on insertion.

How well did you know this?
1
Not at all
2
3
4
5
Perfectly
13
Q

A nurse who is a member of an emergency response team anticipates that several patients with airway obstruction may need a cricothyroidotomy. For which of the following patients would this procedure be appropriate? Select all that apply.

A

Cricothyroidotomy is used in emergencies when endotracheal intubation is either not possible or contraindicated. Examples include airway obstruction from extensive maxillofacial trauma, cervical spine injury, laryngospasm, laryngeal edema after an allergic reaction or extubation, hemorrhage into neck tissue, and obstruction of the larynx.

How well did you know this?
1
Not at all
2
3
4
5
Perfectly
14
Q

The nurse is conducting a secondary survey on a client in the ED. Which action is completed during the secondary survey?

A

Diagnostic and laboratory testing
Explanation:
Diagnostic and laboratory testing is completed during the secondary survey, along with a complete health history, a head-to-toe assessment, insertion or application of monitoring devices, splinting of suspected fractures, cleansing, closure, and dressing of wounds, and performance of other necessary interventions based on the client’s condition. The other interventions are completed during the primary survey.

How well did you know this?
1
Not at all
2
3
4
5
Perfectly
15
Q

A triage nurse in the ED determines that a patient with dyspnea and dehydration is not in a life-threatening situation. What triage category will the nurse choose?

A

Urgent
Explanation:
A basic and widely used triage system that had been in use for many years utilized three categories: emergent, urgent, and nonurgent. In this system, emergent patients had the highest priority, urgent patients had serious health problems but not immediately life-threatening ones, and nonurgent patients had episodic illnesses.

How well did you know this?
1
Not at all
2
3
4
5
Perfectly
16
Q

Which phase of the psychological reaction to rape is characterized by fear and flashbacks?

A

Heightened anxiety phase
Explanation:
During the heightened anxiety phase, the client demonstrates anxiety, hyperalertness, and psychosomatic reactions, in addition to fear and flashbacks. The acute disorganization phase is characterized by shock, disbelief, guilt, humiliation, and anger. The denial phase is characterized by an unwillingness to talk. The reorganization phase occurs when the incident is put into perspective. Some clients never fully recover from rape trauma.

How well did you know this?
1
Not at all
2
3
4
5
Perfectly
17
Q

A client presents to the ED after an unsuccessful suicide attempt. The client is diagnosed with an acetaminophen overdose. The nurse anticipates the administration of which medication?

A

N-acetylcysteine
Explanation:
Treatment of acetaminophen overdose includes administration of N-acetylcysteine. Flumazenil is administered in the treatment of nonbarbiturate sedative overdoses. Naloxone is administered in the treatment of narcotic overdoses. Diazepam may be administered to treat uncontrolled hyperactivity in the client with a hallucinogen overdose.

How well did you know this?
1
Not at all
2
3
4
5
Perfectly
18
Q

A nurse is providing disaster care in an event that is known to involve gamma radiation. When admitting victims of the disaster, what should the nurse do to best reduce victims’ risks of injury?

A

The nurse should have victims shower and change clothes and irrigate or wash open wounds with soap and water. Cleansing the skin helps to reduce the transition from external to internal radiologic contamination. Infectious microorganisms are not involved, so chlorhexidine is of no particular benefit. Applying PPE over contaminated clothing could worsen the risk for injury. Adequate ventilation is important, but removal of contaminants is the priority because of the increased risk for injury.

How well did you know this?
1
Not at all
2
3
4
5
Perfectly
19
Q

The nurse is instructing on bioterrorism agents. Which of the following does the nurse emphasize as an agent which is transmitted from person to person?

A

Smallpox
Explanation:
Smallpox is highly contagious and caused by a variola virus. Individuals infected with the botulinum toxin and anthrax are not at risk to others; there are no reports of person to person transmission. Varicella, commonly called the chickenpox, is contagious but not a bioterrorism agent.

How well did you know this?
1
Not at all
2
3
4
5
Perfectly
20
Q

A client with hypervolemia asks the nurse by what mechanism the sodium–potassium pump will move the excess body fluid. What is the nurse’s best answer?

A

Active transport
Explanation:
Active transport is the physiologic pump maintained by the cell membrane that results in the movement of fluid from an area of lower concentration to one of higher concentration. Active transport requires adenosine triphosphate (ATP) for energy. The sodium–potassium pump actively moves sodium against the concentration gradient out of the cell, and fluid follows. Passive osmosis does not require energy for transport. Free flow is the natural transport of water. Passive elimination is a filter process carried out in the kidneys.

How well did you know this?
1
Not at all
2
3
4
5
Perfectly
21
Q

The nurse is caring for four clients on a medical unit. The nurse is most correct to review which client’s laboratory reports first for an electrolyte imbalance?

A

A 52-year-old with diarrhea
Explanation:
Electrolytes are in both intracellular and extracellular water. Electrolyte deficiency occurs from an inadequate intake of food, conditions that deplete water such as nausea and vomiting, or disease processes that cause an excess of electrolyte amounts. The 52-year-old with diarrhea would be the client most likely to have an electrolyte imbalance. The orthopedic client will not likely have an electrolyte imbalance. Myocardial infarction clients will occasionally have electrolyte imbalance, but this is the exception rather than the rule.

How well did you know this?
1
Not at all
2
3
4
5
Perfectly
22
Q

A nurse is providing care to clients who were involved in an explosion and have sustained secondary blast injuries. Which types of injuries would the nurse expect to find? Select all that apply.

A

Secondary-phase blast injuries, which result from debris or shrapnel within the bomb or from the scene, include penetrating trunk, skin, and soft tissue injuries, fractures, and traumatic amputations. Head injuries are related to the primary phase of the blast injury. Crush injuries and exacerbations of pre-existing conditions are related to the quaternary phase of the blast injury.

How well did you know this?
1
Not at all
2
3
4
5
Perfectly
23
Q

Exposure to gamma radiation can be decreased by completing which action?

A

Providing distance from radiation source
Explanation:
Gamma radiation can penetrate clothing and skin. Thick clothes do not provide any kind of protection. Lead blocks radiation, but it is safest to limit exposure and to distance oneself from the source.

How well did you know this?
1
Not at all
2
3
4
5
Perfectly
24
Q

A nurse is providing care to a client who has been exposed to phosgene vapor. Which nursing diagnosis would the nurse identify as the priority?

A

Impaired gas exchange related to destruction of the pulmonary membrane
Explanation:
Phosgene vapor is a pulmonary agent that destroys the pulmonary membrane leading to pulmonary edema, with shortness of breath. Therefore, impaired gas exchange would be the priority. Impaired skin integrity would be appropriate for exposure to a vesicant. Disturbed sensory perception, visual would be appropriate for a client exposed to a nerve agent. Decreased cardiac output would be appropriate for a client exposed to a blood agent, such as cyanide, which inhibits aerobic metabolism.

How well did you know this?
1
Not at all
2
3
4
5
Perfectly
25
Q

A client is brought to the ED by family members who tell the nurse that the client has been exhibiting paranoid, agitated behavior. What should the nurse do when interacting with this client?

A

Give the client honest answers about likely treatment.
Explanation:
The nurse should offer appropriate and honest explanations in order to foster rapport and trust. Confinement is likely to cause escalation, as is touching the client. The nurse should not normally engage in trying to convince the client that his or her fears are unjustified, as this can also cause escalation.

How well did you know this?
1
Not at all
2
3
4
5
Perfectly
26
Q

A client presents to the ED with a stab wound to the abdomen following an assault. It is suspected that the client has an injury to the pancreas. Which laboratory study is used to detect pancreatic injury?

A

Serum amylase
Explanation:
Serum amylase is analyzed to detect increasing levels, which suggests pancreatic injury or perforation of the gastrointestinal tract. A white blood cell count is done to detect an elevation. Urinalysis is done to detect hematuria. A hemoglobin and hematocrit test is done to evaluate trends reflecting the presence or absence of bleeding.

How well did you know this?
1
Not at all
2
3
4
5
Perfectly
27
Q

A client is diagnosed with hypocalcemia and the nurse is teaching the client about symptoms. What symptom would the nurse include in the teaching?

A

tingling sensation in the fingers
Explanation:
Tingling or numbness in the fingers is a symptom of hypocalcemia. Flank pain, polyuria, and hypertension are symptoms of hypercalcemia.

How well did you know this?
1
Not at all
2
3
4
5
Perfectly
28
Q

A nurse is providing client teaching about the body’s plasma pH and the client asks the nurse what is the major chemical regulator of plasma pH. What is the best response by the nurse?

A

bicarbonate–carbonic acid buffer system
Explanation:
The major chemical regulator of plasma pH is the bicarbonate–carbonic acid buffer system. The renin–angiotensin–aldosterone system regulates blood pressure. The sodium–potassium pump regulate homeostasis. The ADH-ANP buffer system regulates water balance in the body.

How well did you know this?
1
Not at all
2
3
4
5
Perfectly
29
Q

Which condition leads to chronic respiratory acidosis in older adults?

A

Thoracic skeletal change
Explanation:
Poor respiratory exchange as the result of chronic lung disease, inactivity, or thoracic skeletal changes may lead to chronic respiratory acidosis. Decreased renal function in older adults can cause an inability to concentrate urine and is usually associated with fluid and electrolyte imbalance. A poor appetite, erratic meal patterns, inability to prepare nutritious meals, or financial circumstances may influence nutritional status, resulting in imbalances of electrolytes. Overuse of sodium bicarbonate may lead to metabolic alkalosis.

How well did you know this?
1
Not at all
2
3
4
5
Perfectly
30
Q

The nurse has commenced a transfusion of fresh frozen plasma (FFP) and notes the client is exhibiting symptoms of a transfusion reaction. After the nurse stops the transfusion, what is the next required action?

A

If the nurse suspects a transfusion reaction, the transfusion must be stopped immediately and the nurse’s next action is to ensure the normal saline line is running at a rate that permits administration of IV fluids or medications that are required to treat the reaction. The nurse should ensure IV access is maintained. The “to keep vein open” (TKVO) rate allows the nurse to keep the IV client without the potential to cause fluid volume overload. It would be unsafe for the nurse to remove the peripheral IV because continued access is required for urgent IV administration of medications or fluids to treat the reaction. Obtaining a blood culture at the IV site would be necessary if an infection was suspected. This is not required for a transfusion reaction. Normal saline is the solution of choice when transfusing blood products because there is a risk for incompatibility with all other IV solutions.

How well did you know this?
1
Not at all
2
3
4
5
Perfectly
31
Q

The nurse is analyzing the arterial blood gas (ABG) results of a client diagnosed with severe pneumonia. Which of the following ABG results indicates respiratory acidosis?

A

pH: 7.20, PaCO2: 65 mm Hg, HCO3–: 26 mEq/L
Explanation:
Respiratory acidosis is a clinical disorder in which the pH is less than 7.35–7.40 and the PaCO2 is greater than 40–45 mm Hg and a compensatory increase in the plasma HCO3– occurs. Respiratory acidosis may be either acute or chronic.

The ABG of pH: 7.32, PaCO2: 40 mm Hg, HCO3–: 18 mEq/L indicates metabolic acidosis.

The ABGs of pH: 7.50, PaCO2: 30 mm Hg, and HCO3–: 24 mEq/L indicate respiratory alkalosis.

The ABGs of pH 7.40, PaCO2: 40 mm Hg, and HCO3–: 24 mEq/L indicate a normal result/no imbalance.

How well did you know this?
1
Not at all
2
3
4
5
Perfectly
32
Q

A client with suspected inhalation anthrax is admitted to the emergency department. Which action by the nurse takes the highest priority?

A

Monitor vital signs and oxygen saturation every 15 to 30 minutes.
Explanation:
Monitoring vital signs and oxygen saturation every 15 to 30 minutes takes priority. Suctioning the client as needed to obtain a sputum specimen may be necessary, but assessing the client for changes in his respiratory status takes priority. Assessing intake and output and providing adequate hydration are important steps for liquefying secretions; however, they don’t take priority. Reassuring the client that intubation and mechanical ventilation is temporary is inappropriate. The client may not require intubation and mechanical ventilation; however, if he does, the nurse can’t predict the length of time it may be necessary.

How well did you know this?
1
Not at all
2
3
4
5
Perfectly
33
Q

The nurse is caring for a client exposed to a blistering agent. While the nurse is quickly decontaminating the client by showering and bagging all client clothing, what is the nurse simultaneously assessing for?

A

Respiratory compromise
Explanation:
A person exposed to a blistering agent or vesicant must be decontaminated immediately, with clothing removed and bagged. Irrigation of the victim’s eyes and application of topical analgesia, antibiotics, and lubricants to the skin occur. Simultaneously, the nurse is assessing the respiratory system for airway obstruction because blisters from inhaled toxics can swell obstructing respiratory passages.

How well did you know this?
1
Not at all
2
3
4
5
Perfectly
34
Q

A client is to receive hypotonic IV solution in order to provide free water replacement. Which solution does the nurse anticipate administering?

A

0.45% NaCl
Explanation:
Half-strength saline (0.45%) is hypotonic. Hypotonic solutions are used to replace cellular fluid because it is hypotonic compared with plasma. Another is to provide free water to excrete body wastes. At times, hypotonic sodium solutions are used to treat hypernatremia and other hyperosmolar conditions. Lactated Ringer solution and normal saline (0.9% NaCl) are isotonic. A solution that is 5% NaCl is hypertonic.

How well did you know this?
1
Not at all
2
3
4
5
Perfectly
35
Q

A nurse is reviewing a report of a client’s routine urinalysis. Which value requires further

A

Urine pH of 3.0
Explanation:
Normal urine pH is 4.5 to 8; therefore, a urine pH of 3.0 is abnormal and requires further investigation. Urine specific gravity normally ranges from 1.010 to 1.025, making this client’s value normal. Normally, urine contains no protein, glucose, ketones, bilirubin, bacteria, casts, or crystals. Red blood cells should measure 0 to 3 per high-power field; white blood cells, 0 to 4 per high-power field. Urine should be clear, with color ranging from pale yellow to deep amber.

How well did you know this?
1
Not at all
2
3
4
5
Perfectly
36
Q

Inhalation of anthrax mimics which disease process?

A

Flu
Explanation:
Anthrax symptoms mimic those of the flu, and usually treatment is sought only when the second stage of severe respiratory distress occurs. Burns occur with sulfur mustard. Bronchospasm can occur with phosgene or chlorine. Respiratory distress may occur with cyanide.

How well did you know this?
1
Not at all
2
3
4
5
Perfectly
37
Q

A nurse is caring for a client who has arrived at the emergency department in shock. The nurse intervenes based on the knowledge that which of the following is the most common cause of shock?

A

Hypovolemia
Explanation:
Types of shock include cardiogenic, neurogenic, anaphylactic, and septic. Of these, the most common cause is hypovolemia.

How well did you know this?
1
Not at all
2
3
4
5
Perfectly
38
Q

A 6-year-old is admitted to the ED after being rescued from a pond after falling through the ice while ice skating. What action should the nurse perform while rewarming the client?

A

Ensuring continuous ECG monitoring
Explanation:
A hypothermic client requires continuous ECG monitoring and assessment of core temperatures with an esophageal probe, bladder, or rectal thermometer. Massage is not performed and bronchodilators would normally be insufficient to meet the client’s respiratory needs.

How well did you know this?
1
Not at all
2
3
4
5
Perfectly
39
Q

A patient with frostbite to both lower extremities from exposure to the elements is preparing to have rewarming of the extremities. What intervention should the nurse provide prior to the procedure?

A

Administer an analgesic as ordered.
Explanation:
During rewarming, an analgesic for pain is administered as prescribed, because the rewarming process may be very painful. To avoid further mechanical injury, the body part is not handled. Massage is contraindicated.

How well did you know this?
1
Not at all
2
3
4
5
Perfectly
40
Q

The nurse received a patient from a motor vehicle accident who is hemorrhaging from a femoral wound. What is the initial nursing action for the control of the hemorrhage?

A

Apply firm pressure over the involved area or artery.
Explanation:
Direct, firm pressure is applied over the bleeding area or the involved artery at a site that is proximal to the wound (Fig. 72-3). Most bleeding can be stopped or at least controlled by application of direct pressure. Otherwise, unchecked arterial bleeding results in death. A firm pressure dressing is applied, and the injured part is elevated to stop venous and capillary bleeding, if possible. If the injured area is an extremity, the extremity is immobilized to control blood loss. A tourniquet is applied to an extremity only as a last resort when the external hemorrhage cannot be controlled in any other way and immediate surgery is not feasible.

How well did you know this?
1
Not at all
2
3
4
5
Perfectly
41
Q

The nurse is caring for a client who is being prepared for the placement of a central intravenous line. The nurse recognizes this client requires this type of intravenous access for which reason?

A

The client requires total parenteral nutrition
Explanation:
For a patient who requires total parenteral nutrition (TPN), a central intravenous line is required due to the length of time the client will require the infusion as well as the nature of the solution itself. A large vein is required to safely infuse TPN. For this reason, a central line is needed. A peripheral intravenous line is safe to used when IV access is required under six days. Beyond this time, either a new peripheral IV will need to be inserted. If it is known in advance that IV treatment will last beyond six days, the client’s health care provider will order the placement of a central intravenous line. Intravenous antibiotics can be administered peripherally unless the course is longer than six days. D5W is an intravenous solution that can be administered either peripherally or centrally. The nature of this IV solution would not determine which type of IV access the client requires.

How well did you know this?
1
Not at all
2
3
4
5
Perfectly
42
Q

A nurse is performing a primary survey of a client brought to the emergency department. Which of the following would the nurse include? Select all that apply.

A

The primary survey addresses airway, breathing, circulation, and disability. The nurse would establish a patent airway, provide adequate ventilation, evaluate and restore cardiac output, and determine neurologic disability by assessing neurologic function. Obtaining a complete health history and applying monitoring devices are activities involved with the secondary survey.

How well did you know this?
1
Not at all
2
3
4
5
Perfectly
43
Q

Which category of triage encompasses clients with serious health problems that are not immediately life threatening?

A

Urgent
Explanation:
Urgent clients have serious health problems that not immediately life threatening. They must be seen within 1 hour. Emergent clients have the highest priority with life-threatening conditions and they must be seen immediately. Nonurgent clients have episodic illness that can be addressed within 24 hours without increased morbidity. Fast-track clients require simple first aid or basic primary care and may be treated in the ED or safely referred to a clinic or physician’s office.

How well did you know this?
1
Not at all
2
3
4
5
Perfectly
44
Q

A nurse is completing her annual cardiopulmonary resuscitation training. The class instructor tells her that a client has fallen off a ladder and is lying on his back; he is unconscious and isn’t breathing. What maneuver should the nurse use to open his airway?

A

Jaw-thrust
Explanation:
If a neck or spine injury is suspected, the jaw-thrust maneuver should be used to open the client’s airway. To perform this maneuver, the nurse should position herself at the client’s head and rest her thumbs on his lower jaw, near the corners of his mouth. She should then grasp the angles of his lower jaw with her fingers and lift the jaw forward. The head tilt-chin lift maneuver is used to open the airway when a neck or spine injury isn’t suspected. To perform this maneuver the nurse places two fingers on the chin and lifts while pushing down on the forehead with the other hand. The abdominal thrust is used to relieve severe or complete airway obstruction caused by a foreign body. The Seldinger maneuver is a method of percutaneous introduction of a catheter into a vessel.

How well did you know this?
1
Not at all
2
3
4
5
Perfectly
45
Q

A client is experiencing respiratory insufficiency and cannot maintain spontaneous respirations. The nurse suspects that the health care provider will perform which of the following actions?

A

Perform endotracheal intubation.
Explanation:
Endotracheal tubes are used in cases when the client cannot be ventilated with an oropharyngeal airway, which is used in clients who are breathing spontaneously. The jaw thrust maneuver does not establish an airway and cricothyroidotomy would be performed as a last resort.

How well did you know this?
1
Not at all
2
3
4
5
Perfectly
46
Q

A client is admitted to the ED after being involved in a motor vehicle accident. The client has multiple injuries. After establishing an airway and adequate ventilation, the ED team should prioritize what aspect of care?

A

Control the client’s hemorrhage.
Explanation:
After establishing airway and ventilation, the team should evaluate and restore cardiac output by controlling hemorrhage. This must precede neurologic assessments and treatment of skeletal injuries.

How well did you know this?
1
Not at all
2
3
4
5
Perfectly
47
Q

A patient brought to the ED by the rescue squad after getting off a plane at the airport is complaining of severe joint pain, numbness, and an inability to move the arms. The patient was on a diving vacation and went for a last dive this morning before flying home. What is a priority action by the nurse?

A

Ensure a patent airway and that the patient is receiving 100% oxygen.
Explanation:
Decompression sickness, also known as “the bends,” occurs in patients who have engaged in diving (lake/ocean diving), high-altitude flying, or flying in commercial aircraft within 24 hours after diving. Signs and symptoms include joint or extremity pain, numbness, hypesthesia, and loss of range of motion. A patent airway and adequate ventilation are established before all other interventions, as described previously, and 100% oxygen is administered throughout treatment and transport.

How well did you know this?
1
Not at all
2
3
4
5
Perfectly
48
Q

A patient is brought to the emergency department. Assessment reveals that the patient is lethargic and diaphoretic and complaining of right upper quadrant pain. Acetaminophen toxicity is suspected and an acetaminophen level is drawn. Which result would the nurse interpret as indicating toxicity for the patient if he weighs 70 kg?

A

10,500 mg
Explanation:
An acetaminophen level greater than or equal to 140 mg/kg would be considered toxic. For a patient weighing 70 kg, the toxic level would be 9800 mg. A level of 10,500 mg would be greater, thus indicating toxicity.

How well did you know this?
1
Not at all
2
3
4
5
Perfectly
49
Q

The nurse is administering 100% oxygen to a patient with carbon monoxide poisoning and obtains a carboxyhemoglobin level. Which level would the nurse interpret as indicating that oxygen therapy can be discontinued?

A

4%
Explanation:
Oxygen is administered until the carboxyhemoglobin level is less than 5%

How well did you know this?
1
Not at all
2
3
4
5
Perfectly
50
Q

A patient has undergone a diagnostic peritoneal lavage. The nurse interprets which result as indicating a positive test?

A

Evidence of feces
Explanation:
A diagnostic peritoneal lavage is considered positive if there is bile, feces, or food in the specimen, a red blood cell count greater than 100,000/mm3, and a white blood cell count greater than 500/mm3.

How well did you know this?
1
Not at all
2
3
4
5
Perfectly
51
Q

A client undergoes a total abdominal hysterectomy. When assessing the client 10 hours later, the nurse identifies which finding as an early sign of shock?

A

Increasing heart rate
Explanation:
Early in shock, heart rate increases. Inadequate tissue perfusion causes pale, cool, clammy skin (not pale, warm, dry skin). In the early stages of shock, the client’s heart rate will become elevated above normal. In early shock the client’s blood pressure will remain normal, but as shock progresses the mechanisms that regulate blood pressure will not be able to compensate.

How well did you know this?
1
Not at all
2
3
4
5
Perfectly
52
Q

A client is brought to the emergency department after being involved in a motor vehicle collision. Which of the following would lead the nurse to suspect internal bleeding?

A

Delayed capillary refill
Explanation:
If a client exhibits tachycardia, falling blood pressure, thirst, apprehension, cool moist skin, or delayed capillary refill, internal bleeding should be suspected.

How well did you know this?
1
Not at all
2
3
4
5
Perfectly
53
Q

The ED staff work collaboratively and follow the ABCDE method to establish and treat health priorities effectively in a client experiencing a trauma. Which action is completed by the nurse when implementing the “D” element of this method?

A

Assessing the client’s Glasgow Coma Scale score
Explanation:
The primary survey focuses on stabilizing life-threatening conditions. The ED staff work collaboratively and follow the ABCDE (airway, breathing, circulation, disability, exposure) method. While implementing the D element, the nurse determines neurologic disability by assessing neurologic function using the Glasgow Coma Scale and performing a motor and sensory evaluation of the spine. A quick neurologic assessment may be performed using the AVPU mnemonic: A, alert: is the client alert and responsive? V, verbal: does the client respond to verbal stimuli? P, pain: does the client respond only to painful stimuli? U, unresponsive: is the client unresponsive to all stimuli, including pain? The other interventions are not included in this element of the primary survey.

How well did you know this?
1
Not at all
2
3
4
5
Perfectly
54
Q

A high school football player is brought to the emergency department after collapsing at practice in extremely hot and humid weather. Which of the following would lead the nurse to suspect that the client is experiencing heat stroke?

A

Delirium
Explanation:
Manifestations of heat stroke include a temperature of 105 degrees F or greater (40.5 degrees C or greater), anhidrosis (absence of sweating), central nervous system dysfunction (bizarre behavior, delirium, confusion, or coma), hot, dry skin, tachycardia, tachypnea, and hypotension.

How well did you know this?
1
Not at all
2
3
4
5
Perfectly
55
Q

A client presents to the ED reporting choking on a chicken bone. The client is breathing spontaneously. The nurse applies oxygen and suspects a partial airway obstruction. Which action should the nurse do next?

A

If the client can breathe and cough spontaneously, a partial obstruction should be suspected. The client is encouraged to cough forcefully and to persist with spontaneous coughing and breathing efforts as long as good air exchange exists. There may be some wheezing between coughs. If the client demonstrates a weak, ineffective cough, a high-pitched noise while inhaling, increased respiratory difficulty, or cyanosis, the client should be managed as if there were complete airway obstruction. If the client is unconscious, inspection of the oropharynx may reveal the offending object. X-ray study, laryngoscopy, or bronchoscopy also may be performed. There is no indication that an artificial airway is indicated.

How well did you know this?
1
Not at all
2
3
4
5
Perfectly
56
Q

A nurse is working as a camp nurse during the summer. A camp counselor comes to the clinic after receiving a snakebite on the arm. What is the first action by the nurse?

A

Have the patient lie down and place the arm below the level of the heart.
Explanation:
Initial first aid at the site of the snakebite includes having the person lie down, removing constrictive items such as rings, providing warmth, cleansing the wound, covering the wound with a light sterile dressing, and immobilizing the injured body part below the level of the heart. Airway, breathing, and circulation are the priorities of care. Ice, incision and suction, or a tourniquet is not applied.

How well did you know this?
1
Not at all
2
3
4
5
Perfectly
57
Q

A nurse who is a member of an emergency response team anticipates that several patients with airway obstruction may need a cricothyroidotomy. For which of the following patients would this procedure be appropriate? Select all that apply.

A

Cricothyroidotomy is used in emergencies when endotracheal intubation is either not possible or contraindicated. Examples include airway obstruction from extensive maxillofacial trauma, cervical spine injury, laryngospasm, laryngeal edema after an allergic reaction or extubation, hemorrhage into neck tissue, and obstruction of the larynx.

How well did you know this?
1
Not at all
2
3
4
5
Perfectly
58
Q

A patient is brought to the ED by a friend, who states that a tree fell on the patient’s leg and crushed it while they were cutting firewood. What priority actions should the nurse perform? (Select all that apply.)

A

Major soft tissue injuries are dressed and splinted promptly to control bleeding and pain. If an extremity is injured, it is elevated to relieve swelling and pressure.

How well did you know this?
1
Not at all
2
3
4
5
Perfectly
59
Q

A client presents to the ED following a motor vehicle collision. The client is suspected of having internal hemorrhage. The nurse assesses the client for signs and symptoms of shock. Which are signs and symptoms of shock? Select all that apply.

A

Signs and symptoms of shock include cool, moist skin (resulting from poor peripheral perfusion), decreasing blood pressure, increasing heart rate, delayed capillary refill, and decreasing urine volume.

How well did you know this?
1
Not at all
2
3
4
5
Perfectly
60
Q

The nurse is caring for a client who is agitated and confused. The client is persistently trying to get out of bed and attempted to remove the peripheral IV. The nurse has attempted to re-orient the client; however, this was not effective in de-escalating the client’s agitation. The client yells, “I am going to punch you in the face!” What is the nurse’s next action?

A

Call security personnel to assist
Explanation:
Clients at risk for harming staff members require specific interventions. It is important to first notification of security and administration of the potential for violence. Although medication and physical restraints maybe required, the nurse will not be able to carry out these interventions in a safe manner independently. The nurse should first call for security personnel to assist, all other interventions can be carried out with the support of trained staff. When a client is agitated and has the potential to be violent, they should not be left unattended. Moving out of the client’s view can lead to further agitation for the client and increase the risk for escalating to violence.

How well did you know this?
1
Not at all
2
3
4
5
Perfectly
61
Q

A nurse is establishing a patient’s airway. Which action would the nurse perform first?

A

Repositioning the patient’s head
Explanation:
Establishing an airway may be as simple as repositioning the patient’s head to prevent the tongue from obstructing the pharynx. Subsequent measures would include abdominal thrusts to dislodge a foreign body, head-tilt chin-lift or jaw-thrust maneuver, or insertion of an artificial airway.

How well did you know this?
1
Not at all
2
3
4
5
Perfectly
62
Q

The nurse is caring for a 21-year-old client with a diagnoses of brain death due to injuries sustained in a snowboarding accident. The family has chosen to remove life support measures to allow the client’s death. Upon hearing the family’s decision, what is the nurse’s first action?

A

Request senior medical staff discuss organ donation
Explanation:
Clients who meet the criteria for past health and current diagnosis of brain death are eligible to donate organs to those on the various transplant lists. This places nurses in a difficult position at times due to their simultaneous obligations to care for a particular client and the family while informing organ donation services of a potential donor. When the diagnosis of brain death is made, it is usually up to the senior medical staff and organ procurement services to approach the family about the possibility of organ donation. The nurse’s next best action is to request a senior physician speak to the family in a timely manner so organs can be harvested and made available as needed. Advance directives are typically in place for clients who are older and for whom death may be expected. In this case, the client is young and death is unexpected, advanced directives are not likely and this question would be inappropriate. Although the nurse should assess for interrupted family process, this is not the nurse’s initial action after hearing the family has decided to remove life support. This nursing assessment goes beyond acute care and into the provision of community health services which the family will need throughout their grieving process.

How well did you know this?
1
Not at all
2
3
4
5
Perfectly
63
Q

A client presents to the ED following a chemical burn. The client identifies the source of the burn as white phosphorus. The nurse knows that treatment will include

A

No application of water to the burn.
Explanation:
Water should not be applied to burns from lye or white phosphorus because of the potential for an explosion or for deepening of the burn. All evidence of these chemicals should be brushed off the client before any flushing occurs.

How well did you know this?
1
Not at all
2
3
4
5
Perfectly
64
Q

When assessing a client with suspected carbon monoxide poisoning, which finding would be least reliable?

A

Cherry red skin color
Explanation:
Skin color can range from pink or cherry-red to cyanotic and pale is not a reliable sign. In clients with carbon monoxide poisoning, central nervous system signs such as headache and confusion predominate. Palpitations also may occur.

How well did you know this?
1
Not at all
2
3
4
5
Perfectly
65
Q

The nurse is caring for a patient in the ED who is breathing but unconscious. In order to avoid an upper airway obstruction, the nurse is inserting an oropharyngeal airway. How would the nurse insert the airway?

A

Upside down and then rotated 180 degrees
Explanation:
The nurse should insert the oropharyngeal airway with the tip facing up toward the roof of the mouth until it passes the uvula and then rotate the tip 180 degrees so that the tip is pointed down toward the pharynx. This displaces the tongue anteriorly, and the patient then breathes through and around the airway.`

How well did you know this?
1
Not at all
2
3
4
5
Perfectly
66
Q

A patient in the emergency department is bleeding profusely from numerous large and deep lacerations on the top of his head, right side of his face, and forehead. The nurse determines the need to apply pressure at the appropriate pressure point. The nurse would use which of the following pressure points?

A

The location of the injuries and site of bleeding determine which pressure point to use. In this case, the patient’s bleeding is proximal to the temporal artery; therefore, pressure should be applied to this area, as shown in option A. If the patient was bleeding from the lower portion of the face, pressure would be applied to the facial artery, as in option B. The carotid artery would be used to control bleeding proximal to that area. The subclavian artery would be used to control bleeding proximal to it, such as the lower neck and shoulder area.

How well did you know this?
1
Not at all
2
3
4
5
Perfectly
67
Q

A client present to the ED following a work-related injury to the left hand. The client has an avulsion of the left ring finger. Which correctly describes an avulsion?

A

Tissue tearing away from supporting structures
Explanation:
An avulsion is described as a tearing away of tissue from supporting structures. A laceration is a skin tear with irregular edges and vein bridging. Abrasion is denuded skin. A cut is an incision of the skin with well-defined edges, usually long rather than deep.

How well did you know this?
1
Not at all
2
3
4
5
Perfectly
68
Q

Following a motor vehicle collision, a client is brought to the ED for evaluation and treatment. The client is being assessed for intra-abdominal injuries. The client reports severe left shoulder pain (pain score of 10 on a 1 to 10 scale). The nurse suspects injury to the

A

spleen.
Explanation:
The location of pain can indicate certain types of intra-abdominal injuries. Pain in the left shoulder is common in a client with bleeding from a ruptured spleen, whereas pain in the right shoulder can result from laceration of the liver.

How well did you know this?
1
Not at all
2
3
4
5
Perfectly
69
Q

With which condition should the nurse expect that a decrease in serum osmolality will occur?

A

Kidney failure
Explanation:
Failure of the kidneys results in multiple fluid and electrolyte abnormalities including fluid volume overload. If renal function is so severely impaired that pharmacologic agents cannot act efficiently, other modalities are considered to remove sodium and fluid from the body.

How well did you know this?
1
Not at all
2
3
4
5
Perfectly
70
Q

A 64-year-old client is brought in to the clinic with thirsty, dry, sticky mucous membranes, decreased urine output, fever, a rough tongue, and lethargy. Serum sodium level is above 145 mEq/L. Should the nurse start salt tablets when caring for this client?

A

No, sodium intake should be restricted.
Explanation:
The symptoms and the high level of serum sodium suggest hypernatremia, (excess of sodium). It is necessary to restrict sodium intake. Salt tablets and sodium chloride IV can only worsen this condition but may be required in hyponatremia (sodium deficit). Hypotonic solution IV may be a part of the treatment but not along with the salt tablets.

How well did you know this?
1
Not at all
2
3
4
5
Perfectly
71
Q

The nurse is caring for a client with a serum sodium concentration of 113 mEq/L (113 mmol/L). The nurse should monitor the client for the development of which condition?

A

Confusion
Explanation:
Normal serum concentration ranges from 135 to 145 mEq/L (135–145 mmol/L). Hyponatremia exists when the serum concentration decreases below 135 mEq/L (135 mmol/L). When the serum sodium concentration decreases to <115 mEq/L (<115 mmol/L), signs of increasing intracranial pressure, such as lethargy, confusion, muscle twitching, focal weakness, hemiparesis, papilledema, seizures, and death, may occur. General manifestations of hyponatremia include poor skin turgor, dry mucosa, headache, decreased saliva production, orthostatic fall in blood pressure, nausea, vomiting, and abdominal cramping. Neurologic changes, including altered mental status, status epilepticus, and coma, are probably related to cellular swelling and cerebral edema associated with hyponatremia. Hallucinations are associated with increased serum sodium concentrations.

How well did you know this?
1
Not at all
2
3
4
5
Perfectly
72
Q

A nurse correctly identifies a urine specimen with a pH of 4.3 as being which type of solution?

A

Acidic
Explanation:
Normal urine pH is 4.5 to 8.0; a value of 4.3 reveals acidic urine pH. A pH above 7.0 is considered an alkaline or basic solution. A pH of 7.0 is considered neutral.

How well did you know this?
1
Not at all
2
3
4
5
Perfectly
73
Q

A nurse reviews the arterial blood gas (ABG) values of a client admitted with pneumonia: pH, 7.51; PaCO2, 28 mm Hg; PaO2, 70 mm Hg; and HCO3–, 24 mEq/L. What do these values indicate?

A

Respiratory alkalosis
Explanation:
A client with pneumonia may hyperventilate in an effort to increase oxygen intake. Hyperventilation leads to excess carbon dioxide (CO2) loss, which causes alkalosis — indicated by this client’s elevated pH value. With respiratory alkalosis, the kidneys’ bicarbonate (HCO3–) response is delayed, so the client’s HCO3– level remains normal. The below-normal value for the partial pressure of arterial carbon dioxide (PaCO2) indicates CO2 loss and signals a respiratory component. Because the HCO3– level is normal, this imbalance has no metabolic component. Therefore, the client is experiencing respiratory alkalosis.

How well did you know this?
1
Not at all
2
3
4
5
Perfectly
74
Q

A nurse is conducting an initial assessment on a client with possible tuberculosis. Which assessment finding indicates a risk factor for tuberculosis?

A

The client had a liver transplant 2 years ago.
Explanation:
A history of immunocompromised status, such as that which occurs with liver transplantation, places the client at a higher risk for contracting tuberculosis. Other risk factors include inadequate health care, traveling to countries with high rates of tuberculosis (such as southeastern Asia, Africa, and Latin America), being a health care worker who performs procedures in which exposure to respiratory secretions is likely, and being institutionalized.

How well did you know this?
1
Not at all
2
3
4
5
Perfectly
75
Q

A client in the emergency department reports that he has been vomiting excessively for the past 2 days. His arterial blood gas analysis shows a pH of 7.50, partial pressure of arterial carbon dioxide (PaCO2) of 43 mm Hg, partial pressure of arterial oxygen (PaO2) of 75 mm Hg, and bicarbonate (HCO3–) of 42 mEq/L. Based on these findings, the nurse documents that the client is experiencing which type of acid-base imbalance?

A

Metabolic alkalosis
Explanation:
A pH over 7.45 with a HCO3– level over 26 mEq/L indicates metabolic alkalosis. Metabolic alkalosis is always secondary to an underlying cause and is marked by decreased amounts of acid or increased amounts of base HCO3–. The client isn’t experiencing respiratory alkalosis because the PaCO2 is normal. The client isn’t experiencing respiratory or metabolic acidosis because the pH is greater than 7.35.

How well did you know this?
1
Not at all
2
3
4
5
Perfectly
76
Q

An elderly client takes 40 mg of furosemide twice a day. Which electrolyte imbalance is the most serious adverse effect of diuretic use?

A

Hypokalemia
Explanation:
Hypokalemia (potassium level below 3.5 mEq/L) usually indicates a deficit in total potassium stores. Potassium-losing diuretics, such as furosemide, can induce hypokalemia. Hyperkalemia refers to increased potassium levels. Loop diuretics can bring about lower sodium levels, not hypernatremia. Furosemide does not affect phosphorus levels.

How well did you know this?
1
Not at all
2
3
4
5
Perfectly
77
Q

Which of the following may be the first sign of respiratory acidosis in anesthetized patients?

A

Ventricular fibrillation
Explanation:
Ventricular fibrillation may be the first sign of respiratory acidosis in anesthetized patients. Clinical signs in acute and chronic respiratory acidosis include sudden hypercapnia that can cause increased pulse and respiratory rate, mental cloudiness, dull headache or weakness.

How well did you know this?
1
Not at all
2
3
4
5
Perfectly
78
Q

A nurse caring for a patient with metabolic alkalosis knows to assess for the primary, compensatory mechanism of:

A

b. Increased PaCO2.
Explanation:
The respiratory system compensates by decreasing ventilation to conserve CO2 and increase the PaCO2.
Kusmal breathing- hold onto CO2 which is acidic and will help lower blood pH

How well did you know this?
1
Not at all
2
3
4
5
Perfectly
79
Q

Which of the following arterial blood gas results would be consistent with metabolic alkalosis?

A

Serum bicarbonate of 28 mEq/L
Explanation:
Evaluation of arterial blood gases reveals a pH greater than 7.45 and a serum bicarbonate concentration greater than 26 mEq/L.

How well did you know this?
1
Not at all
2
3
4
5
Perfectly
80
Q

A client comes to the emergency department with status asthmaticus. His respiratory rate is 48 breaths/minute, and he is wheezing. An arterial blood gas analysis reveals a pH of 7.52, a partial pressure of arterial carbon dioxide (PaCO2) of 30 mm Hg, PaO2 of 70 mm Hg, and bicarbonate (HCO3??’) of 26 mEq/L. What disorder is indicated by these findings?

A

Respiratory alkalosis
Explanation:
Respiratory alkalosis results from alveolar hyperventilation. It’s marked by a decrease in PaCO2 to less than 35 mm Hg and an increase in blood pH over 7.45. Metabolic acidosis is marked by a decrease in HCO3? to less than 22 mEq/L, and a decrease in blood pH to less than 7.35. In respiratory acidosis, the pH is less than 7.35 and the PaCO2 is greater than 45 mm Hg. In metabolic alkalosis, the HCO3? is greater than 26 mEq/L and the pH is greater than 7.45.

How well did you know this?
1
Not at all
2
3
4
5
Perfectly
81
Q

A client is diagnosed with hypocalcemia and the nurse is teaching the client about symptoms. What symptom would the nurse include in the teaching?

A

tingling sensation in the fingers
Explanation:
Tingling or numbness in the fingers is a symptom of hypocalcemia. Flank pain, polyuria, and hypertension are symptoms of hypercalcemia.

How well did you know this?
1
Not at all
2
3
4
5
Perfectly
82
Q

Oral intake is controlled by the thirst center, located in which of the following cerebral areas?

A

Hypothalamus
Explanation:
Oral intake is controlled by the thirst center located in the hypothalamus. The thirst center is not located in the cerebellum, brainstem, or thalamus.

How well did you know this?
1
Not at all
2
3
4
5
Perfectly
83
Q

A patient is admitted with severe vomiting for 24 hours as well as weakness and “feeling exhausted.” The nurse observes flat T waves and ST-segment depression on the electrocardiogram. Which potassium level does the nurse observe when the laboratory studies are complete?

A

2.5 mEq/L
Explanation:
Symptoms of hypokalemia (<3.0 mEq/L) include fatigue, anorexia, nausea and vomiting, muscle weakness, polyuria, decreased bowel motility, ventricular asystole or fibrillation, paresthesias, leg cramps, hypotension, ileus, abdominal distention, and hypoactive reflexes. Electrocardiogram findings associated with hypokalemia include flattened T waves, prominent U waves, ST depression, and prolonged PR interval.

How well did you know this?
1
Not at all
2
3
4
5
Perfectly
84
Q

A client seeks medical attention for an acute onset of severe thirst, polyuria, muscle weakness, nausea, and bone pain. Which health history information will the nurse report to the health care provider?

A

Takes high doses of vitamin D

Explanation:
Hypercalcemia can affect many organ systems and symptoms occur when the calcium level acutely rises. Hypercalcemia crisis refers to an acute rise in the serum calcium level. Severe thirst and polyuria are often present. Additional findings include muscle weakness, nausea, and bone pain. Excessive ingestion of vitamin D supplements may cause excessive absorption of calcium. Therefore, the nurse would report this finding to the health care provider. The client’s symptoms are not associated with occasional alcohol intake, a high-fiber eating plan, or the client’s employment status. These findings would not need to be reported.

How well did you know this?
1
Not at all
2
3
4
5
Perfectly
85
Q

Which is the most common cause of symptomatic hypomagnesemia?

A

Alcoholism
Explanation:
Alcoholism is currently the most common cause of symptomatic hypomagnesemia. Intravenous drug use, sedentary lifestyle, and burns are not the most common causes of hypomagnesemia.

How well did you know this?
1
Not at all
2
3
4
5
Perfectly
86
Q

A client experiencing a severe anxiety attack and hyperventilating presents to the emergency department. The nurse would expect the client’s pH value to be

A

7.50
Explanation:
The patient is experiencing respiratory alkalosis. Respiratory alkalosis is a clinical condition in which the arterial pH is >7.45 and the PaCO2 is <38 mm Hg. Respiratory alkalosis is always caused by hyperventilation, which causes excessive “blowing off” of CO2 and, hence, a decrease in the plasma carbonic acid concentration. Causes include extreme anxiety, hypoxemia, early phase of salicylate intoxication, gram-negative bacteremia, and inappropriate ventilator settings.

How well did you know this?
1
Not at all
2
3
4
5
Perfectly
87
Q

The weight of a client with congestive heart failure is monitored daily and entered into the medical record. In a 24-hour period, the client’s weight increased by 2 lb. How much fluid is this client retaining?

A

1 L
Explanation:
A 2-lb weight gain in 24 hours indicates that the client is retaining 1L of fluid.

How well did you know this?
1
Not at all
2
3
4
5
Perfectly
88
Q

Which is considered an isotonic solution?

A

0.9% normal saline
Explanation:
An isotonic solution is 0.9% normal saline (NaCl). Dextran in normal saline is a colloid solution, 0.45% normal saline is a hypotonic solution, and 3% NaCl is a hypertonic solution.

How well did you know this?
1
Not at all
2
3
4
5
Perfectly
89
Q

A priority nursing intervention for a client with hypervolemia involves which of the following?

A

Monitoring respiratory status for signs and symptoms of pulmonary complications.
Explanation:
Hypervolemia, or fluid volume excess (FVE), refers to an isotonic expansion of the extracellular fluid. Nursing interventions for FVE include measuring intake and output, monitoring weight, assessing breath sounds, monitoring edema, and promoting rest. The most important intervention in the list involves monitoring the respiratory status for any signs of pulmonary congestion. Breath sounds are assessed at regular intervals.

How well did you know this?
1
Not at all
2
3
4
5
Perfectly
90
Q

The nurse is caring for a client with a serum potassium concentration of 6.0 mEq/L (6.0 mmol/L) and a fluid volume excess. The client is ordered to receive oral sodium polystyrene sulfonate and furosemide. What other order should the nurse anticipate giving?

A

Discontinue the intravenous lactated Ringer solution.
Explanation:
The lactated Ringer intravenous (IV) fluid is contributing to both the fluid volume excess and the hyperkalemia. In addition to the volume of IV fluids contributing to the fluid volume excess, lactated Ringer solution contains more sodium than daily requirements, and excess sodium worsens fluid volume excess. Lactated Ringer solution also contains potassium, which would worsen the hyperkalemia.

How well did you know this?
1
Not at all
2
3
4
5
Perfectly
91
Q

To evaluate a client for hypoxia, the physician is most likely to order which laboratory test?

A

Arterial blood gas (ABG) analysis
Explanation:
Red blood cell count, sputum culture, total hemoglobin, and ABG analysis all help evaluate a client with respiratory problems. However, ABG analysis is the only test that evaluates gas exchange in the lungs, providing information about the client’s oxygenation status.

How well did you know this?
1
Not at all
2
3
4
5
Perfectly
92
Q

Before seeing a newly assigned client with respiratory alkalosis, a nurse quickly reviews the client’s medical history. Which condition is a predisposing factor for respiratory alkalosis?

A

Extreme anxiety
Explanation:
Extreme anxiety may lead to respiratory alkalosis by causing hyperventilation, which results in excessive carbon dioxide (CO2) loss. Other conditions that may set the stage for respiratory alkalosis include fever, heart failure, injury to the brain’s respiratory center, overventilation with a mechanical ventilator, pulmonary embolism, and early salicylate intoxication. Type 1 diabetes may lead to diabetic ketoacidosis; the deep, rapid respirations occurring in this disorder (Kussmaul respirations) don’t cause excessive CO2 loss. Myasthenia gravis and opioid overdose suppress the respiratory drive, causing CO2 retention, not CO2 loss; this may lead to respiratory acidosis, not alkalosis.

93
Q

A client is being treated in the ICU 24 hours after having a radical neck dissection completed. The client’s serum calcium concentration is 7.6 mg/dL (1.9 mmol/L). Which physical examination finding is consistent with this electrolyte imbalance?

A

Presence of Trousseau sign
Explanation:
After radical neck resection, a client is prone to developing hypocalcemia. Hypocalcemia is defined as a serum value <8.6 mg/dL (<2.15 mmol/L). Signs and symptoms of hypocalcemia include Chvostek sign, which consists of muscle twitching enervated by the facial nerve when the region that is about 2 cm anterior to the earlobe, just below the zygomatic arch, is tapped; and a positive Trousseau sign can be elicited by inflating a blood pressure cuff on the upper arm to about 20 mm Hg above systolic pressure; within 2 to 5 minutes, carpal spasm (an adducted thumb, flexed wrist and metacarpophalangeal joints, and extended interphalangeal joints with fingers together) will occur as ischemia of the ulnar nerve develops. Slurred speech and muscle weakness are signs of hypercalcemia.

94
Q

Which intervention is most appropriate for a client with an arterial blood gas (ABG) of pH 7.5, a partial pressure of arterial carbon dioxide (PaCO2) of 26 mm Hg, oxygen (O2) saturation of 96%, bicarbonate (HCO3-) of 24 mEq/L, and a PaO2 of 94 mm Hg?

A

Instruct the client to breathe into a paper bag.
Explanation:
The ABG results reveal respiratory alkalosis. The best intervention to raise the PaCO2 level would be to have the client breathe into a paper bag. Administering a decongestant, offering fluids frequently, and administering supplemental oxygen wouldn’t raise the lowered PaCO2 level.

95
Q

A nurse is caring for an adult client with numerous draining wounds from gunshots. The client’s pulse rate has increased from 100 to 130 beats per minute over the last hour. The nurse should further assess the client for which of the following?

A

Extracellular fluid volume deficit
Explanation:
Fluid volume deficit (FVD) occurs when the loss extracellular fluid (ECF) volume exceeds the intake of fluid. FVD results from loss of body fluids and occurs more rapidly when coupled with decreased fluid intake. A cause of this loss is hemorrhage.

96
Q

A client is diagnosed with syndrome of inappropriate antidiuretic hormone (SIADH). The nurse should anticipate which laboratory test result?

A

Serum sodium level of 124 mEq/L
Explanation:
In SIADH, the posterior pituitary gland produces excess antidiuretic hormone (vasopressin), which decreases water excretion by the kidneys. This, in turn, reduces the serum sodium level, causing hyponatremia, as indicated by a serum sodium level of 124 mEq/L. In SIADH, the serum creatinine level isn’t affected by the client’s fluid status and remains within normal limits. A hematocrit of 52% and a BUN level of 8.6 mg/dl are elevated. Typically, the hematocrit and BUN level decrease.

97
Q

A patient’s serum sodium concentration is within the normal range. What should the nurse estimate the serum osmolality to be?

A

275–300 mOsm/kg
Explanation:
In healthy adults, normal serum osmolality is 270 to 300 mOsm/kg (Crawford & Harris, 2011c).

98
Q

Which electrolyte is a major cation in body fluid?

A

Potassium
Explanation:
Potassium is a major cation that affects cardiac muscle functioning. Chloride, bicarbonate, and phosphate are anions.

99
Q

The nurse is caring for a client in heart failure with signs of hypervolemia. Which vital sign is indicative of the disease process?

A

Elevated blood pressure
Explanation:
Indicative of hypervolemia is a bounding pulse and elevated blood pressure due to the excess volume in the system. Respirations are not typically affected unless there is fluid accumulation in the lungs. Temperature is not generally affected.

100
Q

The physician has prescribed a hypotonic IV solution for a patient. Which IV solution should the nurse administer?

A

0.45% sodium chloride
Explanation:
Half-strength saline (0.45% sodium chloride) solution is frequently used as an IV hypotonic solution.

101
Q

A nurse is caring for a client admitted with a diagnosis of exacerbation of myasthenia gravis. Upon assessment of the client, the nurse notes the client has severely depressed respirations. The nurse would expect to identify which acid-base disturbance?

A

Respiratory acidosis
Explanation:
Respiratory acidosis is always from inadequate excretion of CO2 with inadequate ventilation, resulting in elevated plasma CO2 concentrations. Respiratory acidosis can occur in diseases that impair respiratory muscles such as myasthenia gravis.

102
Q

The nurse is caring for a client with laboratory values indicating dehydration. Which clinical symptom is consistent with the dehydration?

A

Dark, concentrated urine
Explanation:
Dehydration indicates a fluid volume deficit. Dark, concentrated urine indicates a lack of fluid volume. Adding more fluid would dilute the urine. The other options indicate fluid excess.

103
Q

A client hospitalized for treatment of a pulmonary embolism develops respiratory alkalosis. Which clinical findings commonly accompany respiratory alkalosis?

A

Light-headedness or paresthesia
Explanation:
The client with respiratory alkalosis may complain of light-headedness or paresthesia (numbness and tingling in the arms and legs). Headache, blurry vision, abdominal pain, and diarrhea may accompany respiratory acidosis. Hallucinations and tinnitus rarely are associated with respiratory alkalosis or any other acid-base imbalance.

104
Q

The nurse is caring for a patient with diabetes type I who is having severe vomiting and diarrhea. What condition that exhibits blood values with a low pH and a low plasma bicarbonate concentration should the nurse assess for?

A

Metabolic acidosis
Explanation:
Metabolic acidosis is a common clinical disturbance characterized by a low pH (increased H+ concentration) and a low plasma bicarbonate concentration. Metabolic alkalosis is a clinical disturbance characterized by a high pH (decreased H+ concentration) and a high plasma bicarbonate concentration. Respiratory acidosis is a clinical disorder in which the pH is less than 7.35 and the PaCO2 is greater than 42 mm Hg and a compensatory increase in the plasma HCO3 occurs. Respiratory alkalosis is a clinical condition in which the arterial pH is greater than 7.45 and the PaCO2 is less than 38 mm Hg.

105
Q

Which of the following measurable urine outputs indicates the client is maintaining adequate fluid intake and balance?

A

A patient with a minimal urine output of 30 mL/hour
Explanation:
A client with minimal urine output of 30 mL/hour provides the nurse with the information that the patient is maintaining proper fluid balance. Less then 30 mL/hour of urine output indicates dehydration and possible poor kidney function.

106
Q

A patient with diabetes insipidus presents to the emergency room for treatment of dehydration. The nurse knows to review serum laboratory results for which of the diagnostic indicators?

A

Sodium level of 150 mEq/L
Explanation:
Hypernatremia (normal serum sodium is 135 to 145 mEq/L) is consistent with increased fluid loss and dehydration in diabetes insipidus.

107
Q

A nurse is monitoring a client being evaluated who has a potassium level of 7 mEq/L (mmol/L). Which electrocardiogram changes will the client display?

A

peaked T waves
Explanation:
The earliest changes occur when the serum potassium level is 7 mEq/L (mmol/L). Cardiac tracings include peaked and narrow T waves, ST segment depression, and a shortened QT interval.

108
Q

A client with emphysema is at a greater risk for developing which acid–base imbalance?

A

chronic respiratory acidosis
Explanation:
Respiratory acidosis, which may be either acute or chronic, is caused by excess carbonic acid, which causes the blood pH to drop below 7.35. Chronic respiratory acidosis is associated with disorders such as emphysema, bronchiectasis, bronchial asthma, and cystic fibrosis.

109
Q

A client with excess fluid volume and hyponatremia is in a comatose state. What are the nursing considerations concerning fluid replacement?

A

Administer small volumes of a hypertonic solution.
Explanation:
In clients with normal or excess fluid volume, hyponatremia is usually treated effectively by restricting fluid with clients who are not neurologically impaired. When the serum sodium concentration is overcorrected (exceeding 140 mEq/L) too rapidly or in the presence of hypoxia or anoxia, the client can develop neurological symptoms. However, if neurologic symptoms are severe (e.g., seizures, delirium, coma), or if the client has traumatic brain injury, it may be necessary to administer small volumes of a hypertonic sodium solution with the goal of alleviating cerebral edema. Incorrect use of these fluids is extremely dangerous, because 1 L of 3% sodium chloride solution contains 513 mEq (mmol/L) of sodium and 1 L of 5% sodium chloride solution contains 855 mEq (mmol/L) of sodium. The recommendation for hypertonic saline administration in clients with craniocerebral trauma is between 0.10 to 1.0 mL of 3% saline per kilogram of body weight per hour.

110
Q

A client reports muscle cramps in the calves and feeling “tired a lot.” The client is taking ethacrynic acid (Edecrin) for hypertension. Based on these symptoms, the client will be evaluated for which electrolyte imbalance?

A

hypokalemia
Explanation:
Hypokalemia causes fatigue, weakness, anorexia, nausea, vomiting, cardiac dysrhythmias, leg cramps, muscle weakness, and paresthesias. Many diuretics, such as ethacrynic acid (Edecrin), also waste potassium. Symptoms of hyperkalemia include diarrhea, nausea, muscle weakness, paresthesias, and cardiac dysrhythmias. Signs of hypocalcemia include tingling in the extremities and the area around the mouth and muscle and abdominal cramps. Hypercalcemia causes deep bone pain, constipation, anorexia, nausea, vomiting, polyuria, thirst, pathologic fractures, and mental changes.

111
Q

The nurse is caring for a client who has been diagnosed with chronic obstructive pulmonary disease (COPD) and is experiencing respiratory acidosis. The client asks what is making the acidotic state. What does the nurse identify as the result of the disease process that causes the fall in pH?

A

The lungs are not able to regulate carbonic acid levels.
Explanation:
In clients with chronic respiratory acidosis, the client’s lungs are not able to regulate carbonic acid levels. The increase in carbonic acid leads to acidosis. In COPD, the client is able to breathe in oxygen, and gas exchange can occur, but the lungs’ ability to remove the carbon dioxide from the system is limited. Although individuals with COPD frequently have a history of smoking, ineffective cilia is not the cause of the acidosis.

112
Q

The nurse is assigned to care for a client with a serum phosphorus concentration of 5.0 mg/dL (1.61 mmol/L). The nurse anticipates that the client will also experience which electrolyte imbalance?

A

Hypocalcemia
Explanation:
The client is experiencing an elevated serum phosphorus concentration. Hyperphosphatemia is defined as a serum phosphorus that exceeds 4.5 mg/dL (1.45 mmol/L). Because of the reciprocal relationship between phosphorus and calcium, a high serum phosphorus concentration tends to cause a low serum calcium concentration.

113
Q

Which sign suggests that a client with the syndrome of inappropriate antidiuretic hormone (SIADH) secretion is experiencing complications?

A

Jugular vein distention
Explanation:
SIADH secretion causes antidiuretic hormone overproduction, which leads to fluid retention. Severe SIADH can cause such complications as vascular fluid overload, signaled by jugular vein distention. This syndrome isn’t associated with tetanic contractions. It may cause weight gain and fluid retention (secondary to oliguria).

114
Q

Oncotic pressure refers to the

A

osmotic pressure exerted by proteins.
Explanation:
Oncotic pressure is a pulling pressure exerted by proteins such as albumin. Osmolality refers to the number of dissolved particles contained in a unit of fluid. Osmotic diuresis occurs when urine output increases as a result of excretion of substances such as glucose. Osmotic pressure is the amount of pressure needed to stop the flow of water by osmosis.

115
Q

A client with cancer is being treated on the oncology unit for bilateral breast cancer. The client is undergoing chemotherapy. The nurse notes the client’s serum calcium concentration is 12.3 mg/dL (3.08 mmol/L). Given this laboratory finding, the nurse should suspect that the

A

malignancy is causing the electrolyte imbalance.
Explanation:
The client’s laboratory findings indicate hypercalcemia. Hypercalcemia is defined as a calcium concentration >10.2 mg/dL (>2.6 mmol/L).The most common causes of hypercalcemia are malignancies and hyperparathyroidism. Malignant tumors can produce hypercalcemia through a variety of mechanisms. The client’s calcium level is elevated; there is no indication that the client’s diet is lacking in calcium-rich food products. Hyperaldosteronism is not associated with a calcium imbalance. Alcohol abuse is associated with hypocalcemia.

116
Q

The nurse correctly identifies which data as an example of blood pressure and heart rate measurements in a client with postural hypotension?

A

supine: BP 120/70 mm Hg, HR 70 bpm; sitting: BP 100/55 mm Hg, HR 90 bpm; standing: BP 98/52 mm Hg, HR 94 bpm
Explanation:
Postural (orthostatic) hypotension is a sustained decrease of at least 20 mm Hg in systolic BP or 10 mm Hg in diastolic BP within 3 minutes of moving from a lying or sitting position to a standing position. The following is an example of BP and HR measurements in a client with postural hypotension: supine: BP 120/70 mm Hg, HR 70 bpm; sitting: BP 100/55 mm Hg, HR 90 bpm; standing: BP 98/52 mm Hg, HR 94 bpm. Normal postural responses that occur when a person moves from a lying to a standing position include (1) a HR increase of 5 to 20 bpm above the resting rate; (2) an unchanged systolic pressure, or a slight decrease of up to 10 mm Hg; and (3) a slight increase of 5 mm Hg in diastolic pressure.

117
Q

The nurse cares for a client with clubbing of the fingers and toes. The nurse should complete which action given these findings?

A

Obtain an oxygen saturation level.
Explanation:
Clubbing of the fingers and toes indicates chronic hemoglobin desaturation (decreased oxygen supply) and is associated with congenital heart disease. The nurse should assess the client’s O2 saturation level and intervene as directed. The other assessments are not indicated.

118
Q

After a physical examination, the provider diagnosed a patient with a grade 4 heart murmur. During assessment, the nurse expects to hear a murmur that is:

A

Loud and may be associated with a thrill sound similar to (a purring cat).
Explanation:
Heart murmurs are characterized by location, timing, and intensity. A grading system is used to describe the intensity or loudness of a murmur. A grade 1 is very faint and difficult to describe, whereas a grade 6 is extremely loud. Refer to Box 12-3 in the text for a description of grades 1 to 6.

119
Q

What is the term for the ability of the cardiac muscle to shorten in response to an electrical impulse?

A

contractility
Explanation:
Contractility is the ability of the cardiac muscle to shorten in response to an electrical impulse. Depolarization is the electrical activation of a cell caused by the influx of sodium into the cell while potassium exits the cell. Repolarization is the return of the cell to the resting state, caused by reentry of potassium into the cell while sodium exits the cell. Diastole is the period of ventricular relaxation resulting in ventricular filling.

120
Q

A client has been diagnosed with atrial fibrillation and has been prescribed warfarin therapy. What should the nurse prioritize when providing health education to the client?

A

The need to have regular blood levels drawn
Explanation:
One drawback of warfarin therapy is the need to have blood levels drawn on a regular basis. The medication does not need to be taken on an empty stomach, and the client does not have to sit upright. Adequate fluid intake is useful in a general way, but the need for fluids is not increased by taking warfarin.

121
Q

The nurse is caring for a client who is being discharged after insertion of a permanent pacemaker. The client, an avid tennis player, is scheduled to play in a tournament in 1 week. What is the best advice the nurse can give related to this activity?

A

“You will need to cancel this activity; you must restrict arm movement above your head for 2 weeks.”
Explanation:
It is important to restrict movement of the arm until the incision heals. The client should not raise the arm above the head for 2 weeks afterward to avoid dislodging the leads. The client must avoid contact sports (e.g., basketball, football, hockey).

122
Q

What does decreased pulse pressure reflect?

A

reduced stroke volume
Explanation:
Decreased pulse pressure reflects reduced stroke volume and ejection velocity or obstruction to blood flow during systole. Increased pulse pressure would indicate reduced distensibility of the arteries, along with bradycardia.

123
Q

The nurse cares for a client prescribed warfarin orally. The nurse reviews the client’s prothrombin time (PT) level to evaluate the effectiveness of the medication. Which laboratory values should the nurse also evaluate?

A

international normalized ratio (INR)
Explanation:
The INR, reported with the PT, provides a standard method for reporting PT levels and eliminates the variation of PT results from different laboratories. The INR, rather than the PT alone, is used to monitor the effectiveness of warfarin. The therapeutic range for INR is 2 to 3.5, although specific ranges vary based on diagnosis. The other laboratory values are not used to evaluate the effectiveness of warfarin.

124
Q

While the nurse is preparing a client for a cardiac catheterization, the client states that they have allergies to seafood. Which of the following medications may the nurse give prior to the procedure?

A

Methylprednisolone
Explanation:
Prior to cardiac catheterization, the patient is assessed for previous reactions to contrast agents or allergies to iodine-containing substances, as some contrast agents contain iodine. If allergic reactions are of concern, antihistamines or methylprednisolone (Solu-Medrol) may be administered to the patient before angiography is performed. Furosemide, Lorazepam, and Phenytoin do not counteract allergic reactions.

125
Q

The nursing student asks the nurse how to tell the difference between ventricular tachycardia and ventricular fibrillation on an electrocardiogram strip. What is the best response?

A

“Ventricular fibrillation is irregular with undulating waves and no QRS complex. Ventricular tachycardia is usually regular and fast, with wide QRS complexes.”
Explanation:
Ventricular fibrillation is irregular with undulating waves and no QRS complex, while ventricular tachycardia is usually regular and fast with wide QRS complexes. The rhythms look different on the electrocardiogram strip. The QRS is wide and bizarre or undefined in ventricular fibrillation. The P-R interval is not present in the ventricular dysrhythmias.

126
Q

The nurse is reviewing the morning laboratory test results for a client with cardiac problems. Which finding is a priority to report to the healthcare provider?

A

K+ 3.1 mEq/L
Explanation:
All laboratory levels are within normal limits except for the K+, which is low. A low K+ level can cause ventricular tachycardia or fibrillation.

127
Q

During an initial assessment, the nurse measures the client’s apical pulse and compares it to the peripheral pulse. The difference between the two is known as pulse:

A

deficit.
Explanation:
To determine the pulse deficit, one nurse counts the heart rate through auscultation at the apex while a second nurse simultaneously palpates and counts the radial pulse for a full minute. The difference, if any, is the pulse deficit. Pulse rhythm is the pattern of the pulsations and the pauses between them. Pulse volume is described as feeling full, weak, or thready, meaning barely palpable. The pulse quality refers to its palpated volume.

128
Q

A client asks the nurse what causes the heart to be an effective pump. The nurse informs the client that this is due to the:

A

inherent rhythmicity of cardiac muscle tissue.
Explanation:
Cardiac rhythm refers to the pattern (or pace) of the heartbeat. The conduction system of the heart and the inherent rhythmicity of cardiac muscle produce a rhythm pattern, which greatly influences the heart’s ability to pump blood effectively.

129
Q

The nurse is caring for a patient with a diagnosis of pericarditis. Where does the nurse understand the inflammation is located?

A

The thin fibrous sac encasing the heart
Explanation:
The heart is encased in a thin, fibrous sac called the pericardium, which is composed of two layers. Inflammation of this sac is known as pericarditis.

130
Q

For both outpatients and inpatients scheduled for diagnostic procedures of the cardiovascular system, the nurse performs a thorough initial assessment to establish accurate baseline data. Which data is necessary to collect if the client is experiencing chest pain?

A

description of the pain
Explanation:
If the client is experiencing chest pain, a history of its location, frequency, and duration is necessary. A description of the pain is also needed, including if it radiates to a particular area, what precipitates its onset, and what brings relief. The nurse weighs the client and measures vital signs. The nurse may measure blood pressure in both arms and compare findings. The nurse assesses apical and radial pulses, noting rate, quality, and rhythm. The nurse also checks peripheral pulses in the lower extremities.

131
Q

A client is admitted to the emergency department reporting chest pain and shortness of breath. The nurse notes an irregular rhythm on the bedside electrocardiograph monitor. The nurse counts 9 RR intervals on the client’s 6-second rhythm tracing. The nurse correctly identifies the client’s heart rate as

A

90 bpm.
Explanation:
An alternative but less accurate method for estimating heart rate, which is usually used when the rhythm is irregular, is to count the number of RR intervals in 6 seconds and multiply that number by 10. The RR intervals are counted, rather than QRS complexes, because a computed heart rate based on the latter might be inaccurately high. The same methods may be used for determining atrial rate, using the PP interval instead of the RR interval. In this instance, 9 × 10 = 90.

132
Q

The nurse is providing teaching to a client with an implanted cardiac device. Which client statement indicates that teaching has been effective?

A

“I will not place my cell phone in my chest pocket.”

Explanation:
The implantable cardioverter defibrillator (ICD) is an electronic device that detects and terminates life-threatening episodes of tachycardia or fibrillation, especially those that are ventricular in origin. An ICD has a generator about the size of a pack of chewing gum that is implanted in a subcutaneous pocket, usually in the upper chest wall. Because of this, electronic devices should not be placed near the implanted generator as this could cause electromagnetic interference. There is no reason for the client to stop using the microwave oven. Since the MRI is a large magnetic field, MRIs should not be done in the future. A client is not restriced from flying due to having a pacemaker..

133
Q

A client experiences a faster-than-normal heart rate when drinking more than two cups of coffee in the morning. What does the nurse identify on the electrocardiogram as an indicator of sinus tachycardia?

A

heart rate of 118 bpm
Explanation:
The sinus node creates an impulse at a faster-than-normal rate. The PR interval of 0.1 seconds, QRS duration of 0.16 seconds and Q wave of 0.04 seconds are consistent with a normal sinus rhythm. Sinus tachycardia occurs when the heart rate is over 100 bpm.

134
Q

The nurse reviews a client’s lab results and notes a serum calcium level of 7.9 mg/dL. It is most appropriate for the nurse to monitor the client for what condition?

A

Impaired myocardial contractility
Explanation:
Normal serum calcium is 8.9 to 10.3 mg/dL. A reading of 7.9 is below normal. Hypocalcemia is associated with slow nodal functioning and impaired myocardial contractility, which can increase the risk of heart failure.

135
Q

The nurse knows that what PR interval presents a first-degree heart block?

A

0.24 seconds
Explanation:
In adults, the normal range for the PR is 0.12 to 0.20 seconds. A PR internal of 0.24 seconds would indicate a first-degree heart block.

136
Q

The nurse is caring for a client who is scheduled for a transesophageal echocardiogram. What nursing intervention is a priority after the procedure?

A

Keep the head of the bed elevated 45 degrees and keep NPO until return of the gag reflex.
Explanation:
During the recovery period, the client must have the head of the bed elevated 45 degrees to avoid aspiration. The nurse should restrict food and fluids until the return of the gag reflex and the client is fully awake and alert. There will be no puncture site after an transesophageal echocardiogram. There is no need to turn the client on the right side or watching for bleeding from the esophagus. There are no anticoagulants given during this procedure, so bloody stools or urine should not occur.

137
Q

The nurse is assessing a patient with a probable diagnosis of first-degree AV block. The nurse is aware that this dysrhythmia is evident on an ECG strip by what indication?

A

Delayed conduction, producing a prolonged PR interval
Explanation:
First-degree AV block may occur without an underlying pathophysiology, or it can result from medications or conditions that increase parasympathetic tone. It occurs when atrial conduction is delayed through the AV node, resulting in a prolonged PR interval.

138
Q

The nurse is reviewing the results of the patient’s echocardiogram and observes that the ejection fraction is 35%. The nurse anticipates that the patient will receive treatment for what condition?

A

Heart failure
Explanation:
An ejection fraction of less than 40% indicates that the patient has decreased left ventricular function and likely requires treatment for heart failure.

139
Q

The client is admitted for a scheduled cardiac catheterization. On the morning of the procedure, while assessing the client’s morning laboratory values, the nurse notes a blood urea nitrogen (BUN) of 34 mg/dL and a creatinine of 4.2 mg/dL. What priority reason will the nurse notify the healthcare provider?

A

The client is at risk for renal failure due to the contrast agent that will be given during the procedure.
Explanation:
The contrast medium must be excreted by the kidneys. If there is already a degree of renal impairment, indicated by the increased BUN and creatinine, the risk for contrast agent-induced nephropathy and renal failure is high. Renal impairment is not usually associated with dysrhythmias. The increased BUN and creatinine do not indicate overhydration, but decreased kidney function. The BUN and creatinine levels do not interfere with coagulability or bleeding.

140
Q

Each chamber of the heart has a particular role in maintaining cellular oxygenation. Which chamber is responsible for pumping blood to all the cells and tissues of the body?

A

left ventricle
Explanation:
The left ventricle pumps blood to all the cells and tissues of the body. The left atrium receives oxygenated blood from the lungs. The right ventricle pumps blood to the lungs to be oxygenated. The right atrium receives deoxygenated blood from the venous system.

141
Q

The client asks the nurse what urine output has to do with cardiac function. What is the best response by the nurse?

A

“Poor urine output may indicate inadequate blood flow to the kidneys.”
Explanation:
Urine output is an important indicator of cardiac function. Low urine output is caused by the inability of the heart to generate enough cardiac output, leading to reduced blood flow to the brain and other vital organs such as the kidneys. High urine output may indicate an endocrine problem.

142
Q

A nurse is reviewing laboratory values for a client diagnosed with hyperlipidemia 6 months ago. Which results indicate that the client has been following a therapeutic regimen?

A

High density lipoproteins (HDL) increase from 25 mg/dl to 40 mg/dl.
Explanation:
The goal of treating hyperlipidemia is to decrease total cholesterol and LDL levels while increasing HDL levels. HDL levels should be greater than 35 mg/dl. This client’s increased HDL levels indicate that a therapeutic regimen has been followed. Recommended total cholesterol levels are below 200 mg/dl. LDL levels should be less than 160 mg/dl, or, in clients with known coronary artery disease (CAD) or diabetes mellitus, less than 70 mg/dl. Triglyceride levels should be between 100 and 200 mg/d.

143
Q

A nurse is performing discharge teaching with a client who has an implantable cardioverter defibrillator (ICD) placed. Which client statement indicates effective teaching?

A

“I’ll keep a log of each time my ICD discharges.”
Explanation:
The client stating that he should keep a log of all ICD discharges indicates effective teaching. This log helps the client and physician identify activities that may cause the arrhythmias that make the ICD discharge. He should also record the events right before the discharge. Clients with ICDs should avoid contact sports such as football. They must also avoid magnetic fields, which could permanently damage the ICD. Household appliances don’t interfere with the ICD.

144
Q

Which dysrhythmia has an atrial rate between 250 and 400, with saw-toothed P waves?

A

Atrial flutter
Explanation:
Atrial flutter occurs in the atrium and creates impulses at a regular atrial rate between 250 and 400 times per minute. The P waves are saw-toothed in shape. Atrial fibrillation causes a rapid, disorganized, and uncoordinated twitching of atrial musculature. Ventricular fibrillation is a rapid, disorganized ventricular rhythm that causes ineffective quivering of the ventricles. Ventricular tachycardia is defined as three or more PVCs in a row, occurring at a rate exceeding 100 beats per minute.

145
Q

You are monitoring the results of laboratory tests performed on a client admitted to the cardiac ICU with a diagnosis of myocardial infarction. Which test would you expect to show elevated levels?

A

Enzymes
Explanation:
When tissues and cells break down, are damaged, or die, great quantities of certain enzymes are released into the bloodstream. Enzymes can be elevated in response to cardiac or other organ damage. After an MI, RBCs and platelets should not be elevated. WBCs would only be elevated if there was a bacterial infection present.

146
Q

A client describes chest pain as sharp, substernal, of intermittent duration, and radiating to the arms and back. The client says the pain increases with inspiration and swallowing and is alleviated when sitting upright. What does the nurse suspect the client may be experiencing?

A

pericarditis
Explanation:
Chest pain described as a sharp, substernal, of intermittent duration, and radiating to the arms and back that increases with inspiration and swallowing and is alleviated when sitting upright is pericarditis. Angina pectoris pain is often described as a squeezing, pressure, heaviness, tightness, or pain in the chest. Panic attack pain is not always relieved with sitting upright. A client with dissecting aorta experiences back and abdominal pain not relieved with sitting upright.

147
Q

A nurse provides morning care for a client in the intensive care unit (ICU). Suddenly, the bedside monitor shows ventricular fibrillation and the client becomes unresponsive. After calling for assistance, what action should the nurse take next?

A

Begin cardiopulmonary resuscitation
Explanation:
In the acute care setting, when ventricular fibrillation is noted, the nurse should call for assistance and defibrillate the client as soon as possible. If defibrillation is not readily available, CPR is begun until the client can be defibrillated, followed by advanced cardiovascular life support (ACLS) intervention, which includes endotracheal intubation and administration of epinephrine. Electrical cardioversion is not indicated for a client in ventricular fibrillation.

148
Q

Which dysrhythmia is common in older clients?

A

Sinus bradycardia
Explanation:
Sinus bradycardia and atrial fibrillation are common dysrhythmia’a in older clients. Sinus tachycardia, sinus arrhythmmai, and ventricular tachycardia are not common dysrhythmias in older clients.

149
Q

A client reports light-headedness, chest pain, and shortness of breath. They physician orders tests to ascertain what is causing the client’s problems. Which test is used to identify cardiac rhythms?

A

electrocardiogram
Explanation:
An electrocardiogram is used to identify normal and abnormal cardiac rhythms.

150
Q

The nurse recognizes which as being true of cardioversion?

A

Defibrillator should be set to deliver a shock during the QRS complex.
Explanation:
Cardioversion involves the delivery of a “timed” electrical current. The defibrillator is set to synchronize with the ECG and deliver the impulse during the QRS complex. The synchronization prevents the discharge from occurring during the vulnerable period of repolarization (T wave), which could result in VT or ventricular fibrillation.

151
Q

A client is unconscious on arrival to the emergency department. The nurse in the emergency department identifies that the client has a permanent pacemaker due to which characteristic?

A

“Spike” on the rhythm strip
Explanation:
Confirmation that the client has a permanent pacemaker is the characteristic “spike” identified by a thin, straight stroke on the rhythm strip. The scar on the chest is suggestive of pacer implantation but not definitive. There should be no change in pulse quality, and no vibration under the skin.

152
Q

A monitor technician on the telemetry unit asks a charge nurse why every client whose monitor shows atrial fibrillation is receiving warfarin. Which response by the charge nurse is best?

A

“Warfarin prevents clot formation in the atria of clients with atrial fibrillation.”
Explanation:
Blood pools in the atria of clients with atrial fibrillation. As the blood pools, clots form. These clots can be forced from the atria as the heart beats, placing the client at risk for stroke. Warfarin is ordered in most clients with atrial fibrillation to prevent clot formation and decrease the risk of stroke, not to control heart rate. Digoxin is typically ordered to control heart rate in atrial fibrillation. Atrial fibrillation doesn’t typically progress to a lethal arrhythmia such as ventricular fibrillation.

153
Q

The nurse caring for a client who is suspected of having cardiovascular disease has a stress test ordered. The client has a co-morbidity of multiple sclerosis, so the nurse knows the stress test will be drug-induced. What drug will be used to dilate the coronary arteries?

A

Dobutamine
Explanation:
Drugs such as adenosine (Adenocard), dipyridamole (Persantine), or dobutamine (Dobutrex) may be administered singularly or in combination by the IV route. The drugs dilate the coronary arteries, similar to the vasodilation that occurs when a person exercises to increase the heart muscle’s blood supply. The other options would not dilate the coronary arteries.

154
Q

The nurse is assessing vital signs on a client who is 3 months status post myocardial infarction (MI). While the healthcare provider is examining the client, the client’s spouse approaches the nurse and states “We are too afraid he will have another heart attack, so we just don’t have sex anymore.” What is the nurse’s best response?

A

“The physiologic demands are greatest during orgasm and are equivalent to walking 3 to 4 miles per hour on a treadmill.”
Explanation:
The physiologic demands are greatest during orgasm. The level of activity is equivalent to walking 3 to 4 miles per hour on a treadmill. Sexuality is an important quality of life, so the healthcare provider will be determining when it is safe to have intercourse. Erectile dysfunction may be a side effect of beta-blockers, but other medications may be substituted

155
Q

The nurse is administering a stool softener to a client who experienced a myocardial infarction. The client says, “I had a heart attack; I don’t have a problem with constipation.” What explanation will the nurse use to answer the client’s question?

A

“If you strain to have a bowel movement, you can cause a drop in your heart rate that can be dangerous.”
Explanation:
When straining during defecation, the client bears down, which momentarily may cause the heart to slow and cause fainting or syncope in the client. The stool softener will allow easier pass of stool by increasing the amount of water the stool absorbs in the gut, making the stool softer and easier to pass. The client will not have prescribed limited activity after a myocardial infarction. The nurse needs to explain the medication and not refer the client back to the healthcare provider. Stool softeners do not decrease stress.

156
Q

It is important for a nurse to understand cardiac hemodynamics. For blood to flow from the right ventricle to the pulmonary artery, the following must occur:

A

Right ventricular pressure must be higher than pulmonary arterial pressure.
Explanation:
For the right ventricle to pump blood in need of oxygenation into the lungs via the pulmonary artery, right ventricular pressure must be higher than pulmonary arterial pressure.

157
Q

The nurse is assessing a client taking an anticoagulant. What nursing intervention is most appropriate for a client at risk for injury related to side effects of medication enoxaparin?

A

Report any incident of bloody urine, stools, or both.
Explanation:
The client who takes an anticoagulant, such as a low-molecular-weight heparin, is routinely screened for bloody urine, stools, or both. Clients taking enoxaparin will not need to take calcium supplements or have potassium imbalances related to the medication. The clubbing of fingers may occur with chronic pulmonary diseases.

158
Q

The nurse is caring for a client with second-degree atrioventricular block, Type I with symptomatic bradycardia. What is the most likely medication the nurse will administer?`

A

atropine sulfate
Explanation:
Atropine blocks acetylcholine at parasympathetic neuroeffector sites and blocks vagal stimulation. The client will be treated with an anticholinergic that blocks the effects of the vagal nerve. Atenolol and nadolol are beta blockers that are used for chest pain, myocardial infarction, and hypertension. Diltiazem is a calcium channel blocker used to treat angina or slow the heart rate.

159
Q

The nurse is caring for a client who is displaying a third-degree AV block on the EKG monitor. What is the priority nursing intervention for the client?

A

alerting the healthcare provider of the third-degree heart block
Explanation:
The client may experience low cardiac output with third-degree AV block. The healthcare provider needs to intervene to preserve the client’s cardiac output. Monitoring the blood pressure and heart rate are important, but not a priority. The identification of a code status during a heart block is not appropriate. IV fluids are not helpful if the heart is not perfusing.

160
Q

A client’s chart indicates an S4 heart sound, and is scheduled for a cardiac workup. The nurse is aware that this client may have which cardiac condition?

A

hypertensive heart disease
Explanation:
An S4 sound is often associated with hypertensive heart disease. An S3, although normal in children, is often an indication of heart failure in an adult. A friction rub may cause a rough, grating, or scratchy sound that is indicative of pericarditis. Murmurs and clicks caused by turbulent blood flow through diseased heart valves.

161
Q

The nurse cares for a client with diabetes who is scheduled for a cardiac catheterization. Prior to the procedure, it is most important for the nurse to ask which question?

A

“Are you allergic to shellfish?”
Explanation:
Radiopaque contrast agents are used to visualize the coronary arteries. Some contrast agents contain iodine, and the client is assessed before the procedure for previous reactions to contrast agents or allergies to iodine-containing substances (e.g., seafood). If the client has a suspected or known allergy to the substance, antihistamines or methylprednisolone may be administered before the procedure. Although the other questions are important to ask the client, it is most important to ascertain if the client has an allergy to shellfish.

162
Q

The nurse is caring for a client with a damaged tricuspid valve. The nurse knows that the tricuspid valve is held in place by which of the following?

A

Chordae tendineae
Explanation:
Attached to the mitral and tricuspid valves are cordlike structures known as chordae tendineae, which in turn attach to papillary muscles, two major muscular projections from the ventricles. Atrioventricular tendons, semilunar tendineae, and papillary tendons do not hold the tricuspid valve in place.

163
Q

The nurse reviews discharge instructions with a client who underwent a left groin cardiac catheterization 8 hours ago. Which instructions should the nurse include?

A

“Do not bend at the waist, strain, or lift heavy objects for the next 24 hours.”
Explanation:
The nurse should instruct the client to follow these guidelines: For the next 24 hours, do not bend at the waist, strain, or lift heavy objects if the artery of the groin was used; contact the primary provider if swelling, new bruising or pain from the procedure puncture site, or a temperature of 101°F or more occur. If bleeding occurs, lie down (groin approach) and apply firm pressure to the puncture site for 10 minutes. Notify the primary provider as soon as possible and follow instructions. If there is a large amount of bleeding, call 911. The client should not drive to the hospital.

164
Q

During auscultation of the lungs, what would a nurse note when assessing a client with left-sided heart failure?

A

wheezes with wet lung sounds
Explanation:
If the left side of the heart fails to pump efficiently, blood backs up into the pulmonary veins and lung tissue. For abnormal and normal breath sounds, the nurse auscultates the lungs. With left-sided congestive heart failure, auscultation reveals a crackling sound, wheezes, and gurgles. Wet lung sounds are accompanied by dyspnea and an effort to sit up to breathe. With left-sided congestive heart failure, auscultation does not reveal a high pitched sound.

165
Q

The client with a diagnosis of heart failure reports frequently awakening during the night with the need to urinate. What explanation will the nurse offer to explain the urination?

A

Edema is collected in dependent extremities during the day; at night when the client lays down, it is reabsorbed into the circulation and excreted by the kidneys.
Explanation:
Nocturia is common in patients with heart failure. Fluid collected in dependent areas during the day is reabsorbed into the circulation at night when the client is recumbent. The kidneys excrete more urine with the increased circulating volume. The client’s sleeping position does not cause bladder constriction and increased urination. The client’s blood pressure is not causing more urination. The fluid in the client’s lungs does not move to the kidneys at night.

166
Q

A nurse assessing a client who underwent cardiac catheterization finds the client lying flat on the bed. The client’s temperature is 99.8° F (37.7° C). The client’s blood pressure is 104/68 mm Hg. The client’s pulse rate is 76 beats/minute. The nurse detects weak pulses in the leg distal to the puncture site. Skin on the leg is cool to the touch. The puncture site is dry, but swollen. What is the most appropriate action for the nurse to take?

A

Contact the health care provider and report the findings.
Explanation:
The client is probably developing a hematoma at the puncture site. The decreased pulses, swelling, and cool temperature in the leg are all classic signs that blood flow to that extremity is compromised. The nurse should notify the health care provider immediately to preserve the blood flow in the client’s leg. Documenting findings and checking the client again in 1 hour, slowing the I.V. fluid, and encouraging the client to perform isometric leg exercises aren’t appropriate actions for the nurse to take at this time.

167
Q

The nurse is assessing a patient’s blood pressure. What does the nurse document as the difference between the systolic and the diastolic pressure?

A

Pulse pressure
Explanation:
The difference between the systolic and the diastolic pressures is called the pulse pressure.

168
Q

Which area of the heart that is located at the third intercostal space to the left of the sternum?

A

Erb point
Explanation:
Erb point is located at the third intercostal space to the left of the sternum. The aortic area is located at the second intercostal space to the right of the sternum. The pulmonic area is at the second intercostal space to the left of the sternum. The epigastric area is located below the xiphoid process.

169
Q

The nurse is assessing heart sounds in a patient with heart failure. An abnormal heart sound is detected early in diastole. How would the nurse document this?

A

S3
Explanation:
An S3 (“DUB”) is heard early in diastole during the period of rapid ventricular filling as blood flows from the atrium into a noncompliant ventricle. It is heard immediately after S2. “Lub-dub-DUB” is used to imitate the abnormal sound of a beating heart when an S3 is present.

170
Q

A client in the ICU has a central venous pressure (CVP) line placed. The CVP reading is 10 mm Hg. To what condition does the nurse correlate the CVP reading?

A

Right-sided heart failure
Explanation:
Normal CVP is 2 to 8 mm Hg. A CVP greater that 8 mm Hg indicates hypervolemia or right-sided heart failure. A CVP less than 2 mm Hg indicates a reduction in preload or hypovolemia.

171
Q

The nurse observes a client during an exercise stress test (bicycle). Which finding indicates a positive test and the need for further diagnostic testing?

A

ST-segment changes on the ECG
Explanation:
During the test, the following are monitored: two or more ECG leads for heart rate, rhythm, and ischemic changes; blood pressure; skin temperature; physical appearance; perceived exertion; and symptoms, including chest pain, dyspnea, dizziness, leg cramping, and fatigue. The test is terminated when the target heart rate is achieved or if the client experiences signs of myocardial ischemia. Further diagnostic testing, such as a cardiac catheterization, may be warranted if the client develops chest pain, extreme fatigue, a decrease in blood pressure or pulse rate, serious dysrhythmias or ST-segment changes on the ECG during the stress test. The other findings would not warrant stopping the test.

172
Q

The nurse is caring for a client with an elevated blood pressure and no previous history of hypertension. At 0900, the blood pressure was 158/90 mm Hg. At 0930, the blood pressure is 142/82 mm Hg. The nurse is most correct when relating the fall in blood pressure to which structure?

A

Baroreceptors
Explanation:
Baroreceptor sense pressure in nerve endings in the walls of the atria and major blood vessels. The baroreceptors respond accordingly to raise or lower the pressure. Chemoreceptors are sensitive to pH, CO2, and O2 in the blood. Sympathetic nerve fibers increase the heart rate. The vagus nerve slows the heart rate.

173
Q

The nurse is caring for a geriatric client. The client is ordered Lanoxin (digoxin) tablets 0.125mg daily for a cardiac dysrhythmia. Which of the following assessment considerations is essential when caring for this client?

A

Digoxin level
Explanation:
The action of Digoxin slows and strengthens the heart rate. Assessment of the pulse rate is essential prior to administration in all clients. Due to decreased perfusion common in geriatric clients, toxicity may occur more often. The nurse must monitor Digoxin levels in the body. Monitoring symptoms reflecting cardiac output, activity level, and dyspnea are also important assessment considerations for all clients.

174
Q

The nurse cares for a client in the emergency department who has a B-type natriuretic peptide (BNP) level of 115 pg/mL. The nurse recognizes that this finding is most indicative of which condition?

A

heart failure
Explanation:
A BNP level greater than 100 pg/mL is suggestive of heart failure. Because this serum laboratory test can be quickly obtained, BNP levels are useful for prompt diagnosis of heart failure in settings such as the emergency department. Elevations in BNP can occur from a number of other conditions such as pulmonary embolus, myocardial infarction (MI), and ventricular hypertrophy. Therefore, the healthcare provider correlates BNP levels with abnormal physical assessment findings and other diagnostic tests before making a definitive diagnosis of heart failure.

175
Q

The nurse admits an adult female client with a medical diagnosis of “rule out MI.” The client is very frightened and expresses surprise that a woman would have heart problems. What response by the nurse will be most appropriate?

A

“A woman’s heart is smaller and has smaller arteries that become occluded more easily.”
Explanation:
Because the coronary arteries of a woman are smaller, they become occluded from atherosclerosis more easily. The resting rate, stroke volume, and ejection fraction of a woman’s heart are higher than those of a man. The electrical impulses from the sinoatrial node to the atrioventricular node are not different in the genders.

176
Q

A nurse is checking laboratory values on a client who has crackles in the lower lobes, 2+ pitting edema, and dyspnea with minimal exertion. Which laboratory value does the nurse expect to be abnormal?

A

B-type natriuretic peptide (BNP)
Explanation:
The client’s symptoms suggest heart failure. BNP is a neurohormone that’s released from the ventricles when the ventricles experience increased pressure and stretch, such as in heart failure. A BNP level greater than 51 pg/ml is commonly associated with mild heart failure. As the BNP level increases, the severity of heart failure increases. Potassium levels aren’t affected by heart failure. CRP is an indicator of inflammation. It’s used to help predict the risk of coronary artery disease. There is no indication that the client has an increased CRP. There is no indication that the client is experiencing bleeding abnormalities, such as those seen with an abnormal platelet count.

177
Q

A 52-year-old female patient is going through menopause and asks the nurse about estrogen replacement for its cardioprotective benefits. What is the best response by the nurse?

A

“Current evidence indicates that estrogen replacement is not effective at preventing cardiovascular disease and carries some risks.”
Explanation:
In the past, hormone therapy was routinely prescribed for postmenopausal women with the belief that it would deter the onset and progression of coronary artery disease (CAD). However, based on results from the multisite, prospective, longitudinal Women’s Health Initiative study, the American Heart Association (AHA) no longer recommends the use of hormone therapy to prevent cardiovascular disease. However, hormone replacement therapy has not been shown to be harmful for all women, and it may be a good choice for some women seeking to reduce symptoms of menopause.

178
Q

When caring for a client with dysfunction in the conduction system, at which period would the nurse note that cells are resistant to stimulation?

A

During the refractory period
Explanation:
The refractory period is the time when cells are resistant to electrical stimulation. Repolarization is when the ions realign themselves to wait for an electrical signal. Depolarization occurs during muscle contraction when positive ions move inside the myocardial cell membrane and negative ions move outside. Before an impulse is generated, the cells are in a polarized state.

179
Q

A nurse is caring for a client taking diltiazem for arrhythmias. The nurse knows that diltiazem helps decrease arrhythmias by working during which phase of the cardiac action potential?

A

Phase 0
Explanation:
Diltiazem, a calcium channel blocker, blocks the influx of calcium into the cells during phase 0 of the cardiac action potential. This action causes the sinoatrial node and atrioventricular (AV) node to slow their response times, which results in slowed AV conduction, decreased ventricular depolarization, and arrhythmias. Diltiazem doesn’t work during phase 1, 2, or 3 of the cardiac action potential.

180
Q

The nurse instructor is teaching a group of nursing students about adventitious heart sounds. The instructor explains that auscultation of the heart requires familiarization with normal and abnormal heart sounds. What would the instructor tell these students a ventricular gallop indicates in an adult?

A

Heart failure
Explanation:
A sound that follows S1 and S2 is called an S3 heart sound or a ventricular gallop. An S3, normal in children, often is an indication of heart failure in an adult. An extra sound before S1 is an S4 heart sound, or atrial gallop. An S4 sound often is associated with hypertensive heart disease. A friction rub may cause a rough, grating, or scratchy sound that is an indication of pericarditis or inflammation of the pericardium.

181
Q

The nurse is discussing the cardiac system with a client admitted with heart failure. The client asks “What determines the heart rate?” What is the nurse’s best response?

A

The autonomic nervous system controls the heart rate.
Explanation:
The autonomic nervous system primarily controls the heart rate. When the sympathetic branch is stimulated, heart rate increases. When the parasympathetic branch is stimulated, heart rate decreases. Stroke volume is the amount of blood pumped out of the ventricle with each contraction and depends on three factors: preload, afterload, and contractility.

182
Q

The client’s heart rate is observed to be 140 bpm on the monitor. The nurse knows to monitor the client for what condition?

A

Myocardial ischemia
Explanation:
As heart rate increases, diastolic time is shortened, which may not allow adequate time for myocardial perfusion. As a result, clients are at risk for myocardial ischemia (inadequate oxygen supply) during tachycardias (heart rate greater than 100 bpm), especially clients with coronary artery disease.

183
Q

The nurse measures the pulmonary artery wedge pressure in a client with left ventricular dysfunction. Which action will the nurse take after deflating the balloon tip following pressure measurement?

A

Observe for return of the pulmonary artery systolic and diastolic waveforms.

Explanation:
Pulmonary artery pressure monitoring is used in critical care for assessing left ventricular function, diagnosing the etiology of shock, and evaluating the client’s response to medical interventions. After measuring the pulmonary artery wedge pressure, the nurse ensures that the balloon is deflated and that the catheter has returned to its normal position. This intervention is verified by evaluating the return of the pulmonary artery systolic and diastolic waveform displayed on the bedside monitor. The head of the bed does not need to be lowered nor does the blood pressure need to be measured on both arms after measuring the pulmonary artery wedge pressure. The transducer must be positioned at the phlebostatic axis before the measurement is taken to ensure an accurate reading.

184
Q

Which nursing intervention should the nurse perform when a client with valvular disorder of the heart has a heart rate less than 60 beats/min before administering beta-blockers?

A

Withhold the drug and inform the primary health care provider.
Explanation:
Before administering a beta-blocker, the nurse should monitor the client’s apical pulse. If the heart rate is less than 60 bpm, the nurse should withhold the drug and inform the primary health care provider.

185
Q

The nurse is caring for a client with suspected right-sided heart failure. What would the nurse know that clients with suspected right-sided heart failure may experience?

A

Gradual unexplained weight gain
Explanation:
Clients with right-sided heart failure may have a history of gradual, unexplained weight gain from fluid retention. Left-sided heart failure produces paroxysmal nocturnal dyspnea, which may prompt the client to use several pillows in bed or to sleep in a chair or recliner. Right-sided heart failure does not cause increased perspiration or increased urine output.

186
Q

The nurse is working in a long-term care facility with a group of older adults with cardiac disorders. Why would it be important for the nurse to closely monitor an older adult receiving digitalis preparations for cardiac disorders?

A

Older adults are at increased risk for toxicity.
Explanation:
Older adults receiving digitalis preparations are at increased risk for toxicity because of the decreased ability of the kidneys to excrete the drug due to age-related changes. The margin between a therapeutic and toxic effect of digitalis preparations is narrow. Using digitalis preparations does not increase the risk of cardiac arrests, hyperthyroidism, or asthma.

187
Q

The nurse recognizes which symptom as a classic sign of cardiogenic shock?

A

Restlessness and confusion
Explanation:
Cardiogenic shock occurs when decreased cardiac output leads to inadequate tissue perfusion and initiation of the shock syndrome. Inadequate tissue perfusion is manifested as cerebral hypoxia (restlessness, confusion, agitation).

188
Q

The nurse is assessing a newly admitted client with chest pain. What medical disorder is most likely causing the client to have jugular vein distention?

A

heart failure
Explanation:
Elevated venous pressure, exhibited as jugular vein distention, indicates the heart’s failure to pump. Jugular vein distention isn’t a symptom of abdominal aortic aneurysm or pneumothorax. If severe enough, an MI can progress to heart failure, but an MI alone doesn’t cause jugular vein distention.

189
Q

The nurse is admitting a client with frothy pink sputum. What does the nurse suspect is the primary underlying disorder of pulmonary edema?

A

decreased left ventricular pumping
Explanation:
Pulmonary edema is an acute event that results from heart failure. Myocardial scarring, resulting from ischemia, limits the distensibility of the ventricle, making it vulnerable to demands for increased workload. When the demand on the heart increases, there is resistance to left ventricular filling and blood backs up into the pulmonary circulation. Pulmonary edema quickly develops.

190
Q

A client with left-sided heart failure is in danger of impaired renal perfusion. How would the nurse assess this client for impaired renal perfusion?

A

Assess for elevated blood urea nitrogen levels.
Explanation:
Elevated blood urea nitrogen indicates impaired renal perfusion in a client with left-sided heart failure. Serum sodium levels may be elevated. Reduced urine output or elevated blood potassium levels do not indicate impaired renal perfusion in a client with left-sided heart failure.

191
Q

The nurse is obtaining data on an older adult client. What finding may indicate to the nurse the early symptom of heart failure?

A

Dyspnea on exertion
Explanation:
Left-sided heart failure produces hypoxemia as a result of reduced cardiac output of arterial blood and respiratory symptoms. Many clients notice unusual fatigue with activity. Some find exertional dyspnea to be the first symptom. An increase in urinary output may be seen later as fluid accumulates. Hypotension would be a later sign of decompensating heart failure as well as tachycardia.

192
Q

A client with congestive heart failure is admitted to the hospital after reporting shortness of breath. How should the nurse position the client in order to decrease preload?

A

Head of the bed elevated 45 degrees and lower arms supported by pillows
Explanation:
Preload refers to the degree of stretch of the ventricular cardiac muscle fibers at the end of diastole. The client is positioned or taught how to assume a position that facilitates breathing. The number of pillows may be increased, the head of the bed may be elevated, or the client may sit in a recliner. In these positions, the venous return to the heart (preload) is reduced, pulmonary congestion is alleviated, and pressure on the diaphragm is minimized. The lower arms are supported with pillows to eliminate the fatigue caused by the pull of the client’s weight on the shoulder muscles.

193
Q

The nurse observes a client with an onset of heart failure having rapid, shallow breathing at a rate of 32 breaths/minute. What blood gas analysis does the nurse anticipate finding initially?

A

Respiratory alkalosis
Explanation:
At first, arterial blood gas analysis may reveal respiratory alkalosis as a result of rapid, shallow breathing. Later, there is a shift to metabolic acidosis as gas exchange becomes more impaired.

194
Q

Which New York Heart Association classification of heart failure has a poor prognosis and includes symptoms of cardiac insufficiency at rest?

A

IV
Explanation:
Symptoms of cardiac insufficiency at rest are classified as IV, according to the New York Heart Association Classification of Heart Failure. In class I, ordinary activity does not cause undue fatigue, dyspnea, palpitations, or chest pain. In class II, ADLs are slightly limited. In class III, ADLs are markedly limited.

195
Q

The nurse is admitting a client with heart failure. What client statement indicates that fluid overload was occurring at home?

A

“I’m having trouble going up the steps during the day.”
Explanation:
Difficulty with activities like climbing stairs is an indication of a lessened ability to exercise. Eating small meals and not using salt are usually indicated for clients with heart failure. The client’s assertion about morning being the best time of day is a vague statement.

196
Q

A nurse reviews the client’s medical record and reads in the progress notes that the client has decreased left ventricular function. What assessment will validate the diagnosis?

A

orthopnea
Explanation:
Decreased left ventricular function would be characterized by orthopnea, dyspnea, anorexia, bibasilar rales not cleared with coughing, and resting tachycardia.

197
Q

Which term describes the degree of stretch of the ventricular cardiac muscle fibers at the end of diastole?

A

Preload
Explanation:
Preload is the degree of stretch of the ventricular cardiac muscle fibers at the end of diastole. Afterload is the amount of resistance to ejection of blood from a ventricle. The ejection fraction is the percentage of blood volume in the ventricles at the end of diastole that is ejected during systole. Stroke volume is the amount of blood pumped out of the ventricle with each contraction.

198
Q

Which diagnostic study is usually performed to confirm the diagnosis of heart failure?

A

Echocardiogram
Explanation:
An echocardiogram is usually performed to confirm the diagnosis of heart failure. ECG, serum electrolytes, and a BUN are usually completed during the initial workup.

199
Q

The nurse assessing a client with an exacerbation of heart failure identifies which symptom as a cerebrovascular manifestation of heart failure (HF)?

A

Dizziness
Explanation:
Cerebrovascular manifestations of heart failure stemming from decreased brain perfusion include dizziness, lightheadedness, confusion, restlessness, and anxiety due to decreased oxygenation and blood flow.

200
Q

A patient in severe pulmonary edema is being intubated by the respiratory therapist. What priority action by the nurse will assist in the confirmation of tube placement in the proper position in the trachea?

A

Call for a chest x-ray.
Explanation:
A chest x-ray is always obtained after ET tube placement to confirm that the tube is in the proper position within the trachea.

201
Q

A client with pulmonary edema has been admitted to the ICU. What would be the standard care for this client?

A

BP and pulse measurements every 15 to 30 minutes
Explanation:
Bedside ECG monitoring is standard, as are continuous pulse oximetry, automatic BP, and pulse measurements approximately every 15 to 30 minutes.

202
Q

A client is admitted to the hospital with systolic left-sided heart failure. The nurse knows to look for which assessment finding for this client?

A

Pulmonary congestion
Explanation:
When the left ventricle cannot effectively pump blood out of the ventricle into the aorta, the blood backs up into the pulmonary system and causes congestion, dyspnea, and shortness of breath. All the other choices are symptoms of right-sided heart failure. They are all symptoms of systolic failure.

203
Q

Which dysrhythmia is common in older clients?

A

Sinus bradycardia
Explanation:
Sinus bradycardia and atrial fibrillation are common dysrhythmia’a in older clients. Sinus tachycardia, sinus arrhythmmai, and ventricular tachycardia are not common dysrhythmias in older clients.

204
Q

The nurse is monitoring a patient in the postanesthesia care unit (PACU) following a coronary artery bypass graft, observing a regular ventricular rate of 82 beats/min and “sawtooth” P waves with an atrial rate of approximately 300 beat/min. How does the nurse interpret this rhythm?

A

Atrial flutter
Explanation:
Atrial flutter occurs because of a conduction defect in the atrium and causes a rapid, regular atrial rate, usually between 250 and 400 bpm and results in P waves that are saw-toothed. Because the atrial rate is faster than the AV node can conduct, not all atrial impulses are conducted into the ventricle, causing a therapeutic block at the AV node. This is an important feature of this dysrhythmia. If all atrial impulses were conducted to the ventricle, the ventricular rate would also be 250 to 400 bpm, which would result in ventricular fibrillation, a life-threatening dysrhythmia. Atrial flutter often occurs in patients with chronic obstructive pulmonary disease, pulmonary hypertension, valvular disease, and thyrotoxicosis, as well as following open heart surgery and repair of congenital cardiac defects (Fuster, Walsh et al., 2011).

205
Q

A client has been living with an internal, fixed-rate pacemaker. When checking the client’s readings on a cardiac monitor the nurse notices an absence of spikes. What should the nurse do?

A

Double-check the monitoring equipment.
Explanation:
One of the reasons for lack of pacemaker spikes is faulty monitoring equipment.

206
Q

Which dysrhythmia has an atrial rate between 250 and 400, with saw-toothed P waves?

A

Atrial flutter
Explanation:
Atrial flutter occurs in the atrium and creates impulses at a regular atrial rate between 250 and 400 times per minute. The P waves are saw-toothed in shape. Atrial fibrillation causes a rapid, disorganized, and uncoordinated twitching of atrial musculature. Ventricular fibrillation is a rapid, disorganized ventricular rhythm that causes ineffective quivering of the ventricles. Ventricular tachycardia is defined as three or more PVCs in a row, occurring at a rate exceeding 100 beats per minute.

207
Q

The nurse is assessing a client with crackling breath sounds or pulmonary congestion. What is the cause of the congestion?

A

inadequate cardiac output
Explanation:
Pulmonary congestion occurs and tissue perfusion is compromised and diminished when the heart, primarily the left ventricle, cannot pump blood out of the ventricle effectively into the aorta and the systemic circulation. Ascites is fluid in the abdomen, not a cause of congestion. Hepatomegaly is an enlarged liver, which does not cause crackling breath sounds. Nocturia, or voiding at night, does not cause crackling breath sounds.

208
Q

The nurse witnesses a client experiencing ventricular fibrillation. What is the nurse’s priority action?

A

defibrillation
Explanation:
Advanced cardiac life support recommends early defibrillation for witnessed ventricular fibrillation. A cardioversion is used with a client who has a pulse. Atropine is used for bradycardia and dobutamine is an inotropic medication used to increased cardiac output.

209
Q

Which of the following medication classifications is more likely to be expected when the nurse is caring for a client with atrial fibrillation?

A

Anticoagulant
Explanation:
Clients with persistent atrial fibrillation are prescribed anticoagulation therapy to reduce the risk of emboli formation associated with ineffective circulation. The other options may be prescribed but not expected in most situations.

210
Q

A client’s electrocardiogram reveals an irregular rhythm of 75 bpm with a normal QRS and P wave. The nurse who is caring for the client should anticipate:

A

no immediate treatment.
Explanation:
This client’s ECG suggests sinus dysrhythmia. Sinus dysrhythmia does not cause any significant hemodynamic effect and therefore is not typically treated.

211
Q

A 28-year-old client presents to the emergency department, stating severe restlessness and anxiety. Upon assessment, the client’s heart rate is 118 bpm and regular, the client’s pupils are dilated, and the client appears excitable. Which action should the nurse take next?

A

Question the client about alcohol and illicit drug use.
Explanation:
The client is experiencing sinus tachycardia. Since the client’s findings of tachycardia, dilated pupils, restlessness, anxiety, and excitability can indicate illicit drug use (cocaine), the nurse should question the client about alcohol and illicit drug use. This information will direct the client’s plan of care. Causes of tachycardia include medications that stimulate the sympathetic response, stimulants, and illicit drugs. The treatment goals for sinus tachycardia is usually determined by the severity of symptoms and directed at identifying and abolishing its cause. The other interventions may be implemented, but determining the cause of the tachycardia is essential.

212
Q

A client is prescribed digitalis medication. Which condition should the nurse closely monitor when caring for the client?

A

nausea and vomiting
Explanation:
Digitalis medications are potent and may cause various toxic effects. The nurse should monitor the client for signs of digitalis toxicity, not just during the initial period of therapy, but throughout care management. The most common signs and symptoms include nausea and vomiting which can lead to dehydration and electrolyte imbalance. Symptoms of toxicity do not include vasculitis, flexion contractures, or enlargement of joints.

213
Q

After performing an ECG on an adult client, the nurse reports that the PR interval reflects normal sinus rhythm. What is the PR interval for a normal sinus rhythm?

A

0.12 and 0.2 seconds.
Explanation:
The PR interval is measured from the beginning of the P wave to the beginning of the QRS complex. It measures the time needed for conduction through the AV node before ventricular depolarization. The normal range in adults is 0.12 to 0.2 seconds.

214
Q

A nurse is caring for a client with heart failure. The nurse knows that the client has left-sided heart failure when the client makes which statement?

A

“I sleep on three pillows each night.”
Explanation:
Orthopnea is a classic sign of left-sided heart failure. The client commonly sleeps on several pillows at night to help facilitate breathing. Swollen feet, ascites, and anorexia are signs of right-sided heart failure.

215
Q

The staff educator is teaching a class in arrhythmias. What statement is correct for defibrillation?

A

It is used to eliminate ventricular arrhythmias.
Explanation:
The only treatment for a life-threatening ventricular arrhythmia is immediate defibrillation, which has the exact same effect as cardioversion, except that defibrillation is used when there is no functional ventricular contraction. It is an emergency procedure performed during resuscitation. The client is not sedated but is unresponsive. Defibrillation uses more electrical energy (200 to 360 joules) than cardioversion.

216
Q

The nurse is assessing a patient with a probable diagnosis of first-degree AV block. The nurse is aware that this dysrhythmia is evident on an ECG strip by what indication?

A

Delayed conduction, producing a prolonged PR interval
Explanation:
First-degree AV block may occur without an underlying pathophysiology, or it can result from medications or conditions that increase parasympathetic tone. It occurs when atrial conduction is delayed through the AV node, resulting in a prolonged PR interval.

217
Q

A nurse is assessing a client with congestive heart failure for jugular vein distension (JVD). Which observation is important to report to the physician?

A

JVD is noted 4 cm above the sternal angle.
Explanation:
JVD is assessed with the client sitting at a 45° angle. Jugular vein distention greater than 4 cm above the sternal angle is considered abnormal and is indicative of right ventricular failure.

218
Q

The licensed practical nurse is co-assigned with a registered nurse in the care of a client admitted to the cardiac unit with chest pain. The licensed practical nurse is assessing the accuracy of the cardiac monitor, which notes a heart rate of 34 beats/minute. The client appears anxious and states not feeling well. The licensed practical nurse confirms the monitor reading. When consulting with the registered nurse, which of the following is anticipated?

A

The registered nurse administering atropine sulfate intravenously
Explanation:
The licensed practical nurse and registered nurse both identify that client’s bradycardia. Atropine sulfate, a cholinergic blocking agent, is given intravenously (IV) to increase a dangerously slow heart rate. Lanoxin is not administered when the pulse rate falls under 60 beats/minute. It is dangerous to wait until the pulse rate increases without nursing intervention or administering additional medications until the imminent concern is addressed.

219
Q

A client experiences a faster-than-normal heart rate when drinking more than two cups of coffee in the morning. What does the nurse identify on the electrocardiogram as an indicator of sinus tachycardia?

A

heart rate of 118 bpm
Explanation:
The sinus node creates an impulse at a faster-than-normal rate. The PR interval of 0.1 seconds, QRS duration of 0.16 seconds and Q wave of 0.04 seconds are consistent with a normal sinus rhythm. Sinus tachycardia occurs when the heart rate is over 100 bpm.

220
Q

The nurse is caring for a client with second-degree atrioventricular block, Type I with symptomatic bradycardia. What is the most likely medication the nurse will administer?

A

atropine sulfate
Explanation:
Atropine blocks acetylcholine at parasympathetic neuroeffector sites and blocks vagal stimulation. The client will be treated with an anticholinergic that blocks the effects of the vagal nerve. Atenolol and nadolol are beta blockers that are used for chest pain, myocardial infarction, and hypertension. Diltiazem is a calcium channel blocker used to treat angina or slow the heart rate.

221
Q

A patient who had a myocardial infarction is experiencing severe chest pain and alerts the nurse. The nurse begins the assessment but suddenly the patient becomes unresponsive, no pulse, with the monitor showing a rapid, disorganized ventricular rhythm. What does the nurse interpret this rhythm to be?

A

Ventricular fibrillation
Explanation:
The most common dysrhythmia in patients with cardiac arrest is ventricular fibrillation, which is a rapid, disorganized ventricular rhythm that causes ineffective quivering of the ventricles. No atrial activity is seen on the ECG. The most common cause of ventricular fibrillation is coronary artery disease and resulting acute myocardial infarction. Ventricular fibrillation is always characterized by the absence of an audible heartbeat, a palpable pulse, and respirations.

222
Q

The nurse assesses a client with a heart rate of 120 beats per minute. What are the known causes of sinus tachycardia?

A

hypovolemia
Explanation:
The causes of sinus tachycardia include physiologic or psychological stress (acute blood loss, anemia, shock, hypovolemia, fever, and exercise). Vagal stimulation, hypothyroidism, and digoxin will cause a sinus bradycardia.

223
Q

Which ECG waveform characterizes conduction of an electrical impulse through the left ventricle?

A

The QRS complex represents ventricular depolarization. The P wave is an ECG characteristic reflecting conduction of an electrical impulse through the atria. The PR interval is a component of an ECG tracing reflecting conduction of an electrical impulse through the AV node. The QT interval is an ECG characteristic reflecting the time from ventricular depolarization to repolarization.

224
Q

After evaluating a client for hypertension, a health care provider orders atenolol, 50 mg P.O. daily. Which therapeutic effect should atenolol have in treating hypertension?

A

Decreased cardiac output and decreased systolic and diastolic blood pressure
Explanation:
As a long-acting, selective beta1-adrenergic blocker, atenolol decreases cardiac output and systolic and diastolic blood pressure; however, like other beta-adrenergic blockers, it increases peripheral vascular resistance at rest and with exercise. Atenolol may cause bradycardia, not tachycardia.

225
Q

Positive end-expiratory pressure (PEEP) therapy has which effect on the heart?

A

Reduced cardiac output
Explanation:
PEEP reduces cardiac output by increasing intrathoracic pressure and reducing the amount of blood delivered to the left side of the heart. It doesn’t affect heart rate, but a decrease in cardiac output may reduce blood pressure, commonly causing compensatory tachycardia, not bradycardia. However, the resulting tachycardia isn’t a direct effect of PEEP therapy itself.

226
Q

The nurse is caring for a patient with pleurisy. What symptoms does the nurse recognize are significant for this patient’s diagnosis?

A

Stabbing pain during respiratory movement
Explanation:
When the inflamed pleural membranes rub together during respiration (intensified on inspiration), the result is severe, sharp, knifelike pain. The key characteristic of pleuritic pain is its relationship to respiratory movement. Taking a deep breath, coughing, or sneezing worsens the pain. Pleuritic pain is limited in distribution rather than diffuse; it usually occurs only on one side. The pain may become minimal or absent when the breath is held. It may be localized or radiate to the shoulder or abdomen. Later, as pleural fluid develops, the pain decreases.

227
Q

The nurse is caring for a client with a cardiac arrhythmia whose cardiac function is being continuously monitored. In which order will the nurse analyze the rhythm strip?

A

Correct response:
Determine the ventricular rate.

Determine the ventricular rhythm.

Determine the QRS duration.

Determine consistency of the QRS duration.

Identify the shape of the QRS.

Identify P waves and presence before QRS complexes.

Identify P wave consistency and shape.

Determine the atrial rate.

Determine the atrial rhythm

Determine the PR interval for consistency and number for each QRS complex

Explanation:
When evaluating the cardiac rhythm of a client with an arrhythmia, the ventricular rate should be first determined, followed by the ventricular rhythm. The QRS duration is reviewed next and analyzed for duration, consistency, and shape. This is followed by analysis of the P waves, specifically the location before every QRS complex, and the shape and consistency. Atrial rate and rhythm are next evaluated. This is followed by analysis of the PR interval to determine if they are consistent. The last step is to determine the number of P waves that occur for each QRS complex.

228
Q

The nurse is caring for a client with suspected ARDS with a pO2 of 53. The client is placed on oxygen via face mask and the PO2 remains the same. What does the nurse recognize as a key characteristic of ARDS?

A

Unresponsive arterial hypoxemia
Explanation:
Acute respiratory distress syndrome (ARDS) can be thought of as a spectrum of disease, from its milder form (acute lung injury) to its most severe form of fulminate, life-threatening ARDS. This clinical syndrome is characterized by a severe inflammatory process causing diffuse alveolar damage that results in sudden and progressive pulmonary edema, increasing bilateral infiltrates on chest x-ray, hypoxemia unresponsive to oxygen supplementation regardless of the amount of PEEP, and the absence of an elevated left atrial pressure.

229
Q

A client has been brought to the ED by the paramedics. The client is suspected of having acute respiratory distress syndrome (ARDS). What intervention should the nurse first anticipate?

A

Preparing to assist with intubating the client
Explanation:
A client who has ARDS usually requires intubation and mechanical ventilation. Oxygen by nasal cannula would likely be insufficient. Deep suctioning and nebulizers may be indicated, but the priority is to secure the airway.